Medsurg Final

Pataasin ang iyong marka sa homework at exams ngayon gamit ang Quizwiz!

A nurse is interested in providing care for persons with chronic illnesses. Where would the nurse identify that most chronic illnesses are managed? Hospice care Homeless shelters A community setting An acute care hospital

A community setting

When nurses disagree about the effectiveness of commonly practiced nursing intervention, the best evidence for determining which intervention to use it A.) A systematic review of randomized controlled trials B.) A qualitative research study with a large sample size C.) A methodological internet search using key medical terms D.) Anecdotal evidence retrieved from two or more case studies

A.) A systematic review of randomized controlled trials

An older adult from Nigeria is visiting an adult child. The child tries to encourage the patient to have the testing and treatment for a chronic productive cough. Since the child now believes in American medical treatment, what behavior has occurred? A.) Acculturation B.) Ethnocentrism C.) Race awareness D.) Cultural imposition

A.) Acculturation

The nurse identifies a patient at high risk for health care disparities. What would be included as important nursing actions to reduce healthcare disparities? A.) Active listening B.) Relationship building C.) Taking foreign language classes D.) Participating in cross-cultural education E.) Using standardized evidence-based guidelines

A.) Active listening B.) Relationship building D.) Participating in cross-cultural education E.) Using standardized evidence-based guidelines

The charge nurse evaluating staff performance notes which observed action would be most indicative of stereotyping? A.) Advocating for bronchodilators to be prescribed for all newly admitted black patients B.) Helping a recent immigrant to explore ways to pay for her newly prescribed antihypertensive medications C.) Prioritizing the assessment of obesity when providing care for black and Mexican American woman D.) Arranging for translation services to be provided for a patient who has recently emigrated from Sri Lanka and does not speak English

A.) Advocating for bronchodilators to be prescribed for all newly admitted black patients

What factor has been clearly identified as influencing the future of nursing practice? A.) Aging of the American population and increases in chronic illnesses B.) Increasing birth rates coupled with decreasing average life expectancy C.) Increased awareness of determinants of health and improved self-care D.) Apathy around health behaviors and the relationship of lifestyle to health

A.) Aging of the American population and increases in chronic illnesses

Which actions by the nurse best demonstrate awareness of learning styles and the role the nurse plays in patient and caregiver teaching? A.) Assess patients preferred learning styles before teaching B.) Prioritize the learning style with which the patient is most familiar and comfortable C.) Use materials that appeal to a variety of learning styles, including the styles identified by the patient D.) Provide written instructions to your younger patients while using visual and audio aids when teaching older patients E.) Provide visual and audio aids to younger patients while using written instructions when teaching older patients

A.) Assess patients preferred learning styles before teaching B.) Prioritize the learning style with which the patient is most familiar and comfortable C.) Use materials that appeal to a variety of learning styles, including the styles identified by the patient

What are the four components of cultural competence that nurses should demonstrate in providing nursing care in any care delivery setting? A.) Awareness, knowledge, skill, and encounter B.) Patience, sensitivity, openness, and listening C.) Immersion, transendence, imposition, and attitude D.) Ability, understanding, communication, and empathy

A.) Awareness, knowledge, skill, and encounter

The nurse is performing a general survey as part of the health assessment for a new patient in the pain clinic. Which objective data would the nurse obtain as part of the general survey? A.) Body features and body movements B.) Auscultation of heart and lung sounds C.) 24 hour dietary recall from the patient D.) History of present illness and pain assessment

A.) Body features and body movements

While preparing to interview for employment in a rural hospital, the nurse identifies which chronic illnesses as more prevalent in rural areas than in urban areas of the United States? A.) Cancer and heart disease B.) Sexually transmitted infections C.) Alzheimer's disease and dementia D.) Chronic obstructive pulmonary disease

A.) Cancer and heart disease

When planning care for a patient, the nurse may use a visual diagram of patient problems and interventions to illustrate the relationships among pertinent clinical data. This format is called a A.) Concept map B.) Critical pathway C.) Clinical pathway D.) Nursing care plan

A.) Concept map

A group of nurses has a plan to implement evidence-based practice for the care of patients with pressure injuries. What will this change in practice encompass? A.) Consulting with the wound care and ostomy nurse B.) Nurses expertise and bodies of experience and knowledge C.) The preferences of patients and their particular circumstances D.) The traditions that surround pressure injury practices on the unit E.) Journal articles that address the care of patients with pressure injuries

A.) Consulting with the wound care and ostomy nurse B.) Nurses expertise and bodies of experience and knowledge D.) The traditions that surround pressure injury practices on the unit

A female patient with incontinence arrives early for appointments and social events so there is ready access to the restroom. Which tasks of the chronically ill is the patient demonstrating? (Select all that apply.) A.) Controlling symptoms B.) Preventing social isolation C.) Preventing and managing a crisis D.) Denying the reality of the problem E.) Adjusting to changes in the course of the disease

A.) Controlling symptoms B.) Preventing social isolation C.) Preventing and managing a crisis

Telehealth devices can be used to provide which types of patient care? (Select all that apply) A.) Evaluating weight loss B.) Medication Administration C.) Video assessment of wounds D.) Monitoring peak flow meter results E.) Real time blood pressure assessment

A.) Evaluating weight loss B.) Medication Administration C.) Video assessment of wounds D.) Monitoring peak flow meter results E.) Real time blood pressure assessment

Seniors at a community center ask the nurse to suggest computer websites where they can learn more about health. Which websites would the nurse recommend provide accurate and current health information for these older adults? A.) Government websites B.) Commercial websites C.) Websites designed for health professionals D.) Websites developed by an anonymous person

A.) Government websites

The nurse is caring for a patient who requests a curandero to provide care. What action would the nurse take? A.) Notify the health care provider and obtain the services B.) Ensure availability of cultural educational resources C.) Evaluate the safety and appropriateness of the request D.) Identify stereotypic attitudes towards culture/ethnic groups

A.) Notify the health care provider and obtain the services

Which teaching techniques would be most appropriate to enhance patient learning? A.) Obtain frequent feedback B.) Explain all information in great detail C.) Involve the patient and caregiver in the process D.) Choose the learning format that is most readily available E.) Emphasize relevance of the information to the patients life

A.) Obtain frequent feedback C.) Involve the patient and caregiver in the process E.) Emphasize relevance of the information to the patients life

The adult children of a patient who is Hindu have requested a vegetarian diet for the patient. What is the nurses most appropriate response? A.) Order a vegetarian diet for the patient B.) Assess the familys reason for choosing this particular diet C.) Teach the family about the relationship between protein intake and wound healing D.) Explore the benefits of a low meat, high protein diet with the family and the dietitian

A.) Order a vegetarian diet for the patient

A patient is being prepared for discharge home after a laprascopic cholecystectomy. Which team member can be assigned to complete a discharge assessment and provide patient teaching for care after discharge? A.) RN B.) LPN C.) AP D.) NT

A.) RN

A patient is in the contemplation stage of the behavior change model regarding her tobacco use. Which action would the nurse take? A.) Reinforce the stated need for change B.) Reinforce the positive outcomes of change C.) Increase the awareness of the need for change D.) Help the patient plan to deal with potential relapses

A.) Reinforce the stated need for change

Which patient interventions fall within the RNs scope of practice? A.) Reinserting an IV B.) Assessing lung sounds C.) Obtaining informed consent D.) Providing medication teaching E.) Suctioning a patient every two hours

A.) Reinserting an IV B.) Assessing lung sounds D.) Providing medication teaching E.) Suctioning a patient every two hours

To address patient safety concerns the nurse addresses the National Patient Safety Goals to reduce alarm fatigue by A.) Responding to alarms promptly B.) Decreasing the volume for all alarms C.) Turning off all alarms when in a patients room D.) Not responding to alarms for unassigned patients

A.) Responding to alarms promptly

Which statement by the nurse demonstrates the use of a motivational interviewing technique? A.) Tell me about the concerns you have about changing your diet B.) You have a big weakness for ice cream; we will need to change that C.) I would not want to change my diet, but I would do it so I don't get sicker D.) I feel sorry for you, but you must change your diet if you plan to feel better

A.) Tell me about the concerns you have about changing your diet

The RN is in charge of delegating patient care to the team. What patient care will the RN delegate to the LPN/VN? A.) The day 2 postoperative patient ordered daily dressing changes B.) The patient scheduled oral medications at 0800, 1000, and 1200 C.) The patient awaiting admission to the unit from the emergency department D.) The patient who requires teaching administering insulin for home use E.) The patient who is ambulatory and stable awaiting for the provider

A.) The day 2 postoperative patient ordered daily dressing changes B.) The patient scheduled oral medications at 0800, 1000, and 1200 E.) The patient who is ambulatory and stable awaiting for the provider

The patient reports preferring to learn by watching how things are done rather than reading information or instructions. Which teaching strategy would be best for this patient? A.) Viewing a videorecording B.) Talking about the care with the nurse C.) Reading a book with color pictures demonstrating the skill D.) Using the internet to find blogs written by patients with similar care needs.

A.) Viewing a videorecording

Which question would the nurse ask when assessing the patients understanding of maanging a wound care dressing? A.) What will you do if the dressing becomes soiled? B.) Do you want to watch me change your dressing? C.) Do you understand how to put on a clean dressing? D.) Do you think you can change your dressing at home?

A.) What will you do if the dressing becomes soiled?

A nurse is caring for an adult who sustained a severe traumatic brain injury following a motor vehicle accident. Which type of practice setting will the nurse prepare the patient for on discharge? Assisted living Acute rehabilitation Long-term acute care Skilled nursing facility

Acute rehabilitation

A patient with Alzheimer's disease has increased evidence of dementia and physical deterioration. Which would be the best assistance to recommend to the caregiver spouse who is exhausted? Adult day care Long-term care Home health care Homemaker services

Adult day care

The nurse is performing an assessment for an older adult patient suspected of elder mistreatment. Which assessment findings would lead the nurse to notify social services? (Select all that apply.) Agitation Depression Weight gain Weight loss Hypernatremia

Agitation Depression Weight loss Hypernatremia

A patient is diagnosed with chronic obstructive pulmonary disease (COPD). At what point should the nurse begin to include the patient's spouse in the teaching around the management of the disease? As soon as possible When the patient requests help from the spouse and family When the patient becomes unable to manage symptoms independently After the patient has had the opportunity to adjust to the treatment regimen

As soon as possible

The nurse is developing a community outreach program for patients with limited healthcare access. Which patient would most likely benefit from this program? A.) A 65-year-old upper class Hispanic man living in the city B.) A 78-year-old native american man living on a reservation C.) A 72-year-old black retired attorney living in a condominium D.) A 50 year old middle-class white woman living in a rural area

B.) A 78-year-old native american man living on a reservation

The nurse is developing a plan of care for an older adult patient. What must the nurse include in the plan of care? A.) Patient priorities should be the only focus of care. B.) Additional time related to declining energy reserves. C.) Reduction of disease and problems should be the focus. D.) Tobacco cessation will help the patient cope with other illnesses.

B.) Additional time related to declining energy reserves.

The nurse plans to teach adult patients who are scheduled in the primary care office this week. Which general goal will the nurse plan to meet? A.) Decreased pain B.) Health promotion C.) Disease identification D.) Increased quantity of life

B.) Health promotion

A nurse is providing care for a patient who had a transurethral resection of his prostate this morning. The patient is receiving continuous bladder irrigation and the urinary catheter is now occluded. The nurse contacts the patients health care provider and communicates using the SBAR format. Which statement is a componenet of communication using SBAR? A.) What do you think could be causing the occlusion? B.) I think that we should manually irrigate his catheter C.) What so you know about this patient and his history? D.) Could you please provide some direction for his care?

B.) I think that we should manually irrigate his catheter

The nurse is caring for a spanish speaking patient that speaks no english. What action would the nurse take when using an interpreter for communication with this patient? A.) Ask a family member to interpret B.) Maintain eye contact with the patient C.) Ask fewer questions to prevent fatigue D.) Use as many medical terms as possible

B.) Maintain eye contact with the patient

A Muslim patient is hospitalized during the period of Ramadan and refuses his morning oral medications. What is the priority action by the nurse? A.) Place the medication in food B.) Offer the medication to the patient after sundown C.) Tell the patient that if he does not take the medication now, he will not get better D.) Inform the patient that if he does not take his medication he will have to leave the hospital

B.) Offer the medication to the patient after sundown

During which phase of the nursing process does the nurse establish priorities and determine outcomes for an individual patient? A.) Assessment B.) Planning C.) Implementation D.) Evaluation

B.) Planning

Which patient care delivery model focusing on continuity of care and interprofessional collaboration does the nurse use to plan and coordinate aspects of patient care with other disciplines even if the nurse is absent? A.) Team nursing model B.) Primary nursing model C.) Total patient care model D.) Case management nursing model

B.) Primary nursing model

During an admission history and physical assessment, the patient describes symptoms to the nurse. What type of data would these descriptions be documented as? A.) Objective B.) Subjective C.) Generalized D.) Comprehensive

B.) Subjective

A patient repeatedly asks about the purpose and potential side effects of each oral medication received while in the hospital. How would the nurse best interpret the patients question? A.) The patient has an auditory learning style B.) The patient is identifying his learning needs C.) The patient is exhibiting a high level of health literacy D.) The patient has anxiety related to his diagnosis and treatment

B.) The patient is identifying his learning needs

A woman has sought care for restless legs syndrome, a problem that has been causing her loss of sleep in recent weeks. The woman states that she attributes the problem to inadequate calcium intake, and she has responded by integrating more dairy products into her diet. Which aspects of symptom investigation does the nurse recognize as present in this situation? A.) Quality B.) Timing C.) Severity D.) Radiation E.) Precipitating and palliative

B.) Timing D.) Radiation E.) Precipitating and palliative

Which teaching strategy would be the best choice for a 20-year-old patient? A.) Lectures or books B.) Websites or podcasts C.) Television or pamphlets D.) Role-playing or support groups

B.) Websites or podcasts

Which criterion must a 65-yr-old person meet in order to qualify for Medicare funding? Being entitled to Social Security benefits A documented absence of family caregivers A validated need for long-term residential care A history of failed responses to standard medical treatments

Being entitled to Social Security benefits

A patient is being admitted to the hospital for dyspnea related to bacterial pneumonia. The primary reason the nurse interviews the patient is to obtain A.) Objective data to be used to diagnose a medical condition B.) Objective data while performing the physical assessment C.) A history about the patients past and present state of health D.) A general impression of the patient, including their behavior

C.) A history about the patients past and present state of health

The nurse is supervising assistive personnel (AP) providing hygiene care to older adult residents in a care facility. What behavior indicates further AP education is required? A.) Compassion for weakness and low activity B.) Identification of progressive loss of function C.) A negative attitude based on the age of the patient D.) Providing culturally competent care to older adults

C.) A negative attitude based on the age of the patient

A patient being prepared for a major cardiac surgical procedure tells the nurse she is not religious but has spiritual beliefs. How would the nurse best base their response to the patient? A.) Beliefs about life, death, good, and evil B.) The use of prayer for intercession and thanksgiving C.) A person's effort to find purpose and meaning in life D.) A formal system and beliefs, including worship of God or gods

C.) A person's effort to find purpose and meaning in life

A nurse with an associate or baccalaureate degree who meets licensing requirements is qualified to practice as A.) A nurse practitioner B.) A certified specialist C.) An entry level generalist D.) An advanced practice nurse

C.) An entry level generalist

Which teaching method individualizes learning for a patients unique needs? A.) Using prepackaged learning materials B.) Only teaching the patient "need-to-know" information C.) Asking the patient to choose topics in order of priority D.) Having the patient watch a video and then read a pamphlet

C.) Asking the patient to choose topics in order of priority

The nurse is interested in developing cultural competence. What behaviors would be adopted to do so? A.) Develop a thorough understanding of all ethnic groups B.) Identify one culture of interest and become competent in that culture C.) Have extended contact with a cultural group to enhance understanding of its values and beliefs D.) Understand the specific information about the patients culture so the understanding can be applied to caring for the patient

C.) Have extended contact with a cultural group to enhance understanding of its values and beliefs

A nurse is explaining differences in the incidence, prevalence, mortality rate, and burden of diseases among specific population groups to community volunteers. What concept is the nurse ding describing? A.) Stereotyping B.) Ethnocentrism C.) Health disparities D.) Cultural competence

C.) Health disparities

When the nurse encourages a patient with heart failure to alternate rest and activity periods to reduce cardiac workload, which phase of the nursing process is being used? A.) Diagnosis B.) Planning C.) Implementation D.) Evaluation

C.) Implementation

A patient will need to change a dressing on an open leg wound after discharge. Which strategy would the nurse use to evaluate if the patient is able to perform the dressing change? A.) Have the patient complete the dressing change on a model B.) Ask the patient to write down each step of the dressing change C.) Observe the patient completing the dressing change on himself D.) Provide printed instructions with pictures of how to do his dressing change

C.) Observe the patient completing the dressing change on himself

Which nursing assessment findings are objective data? A.) Earache B.) Dizziness C.) Pitting edema D.) Cyanosis of lips E.) Shortness of breath F.) Hyperactive peristalsis

C.) Pitting edema D.) Cyanosis of lips F.) Hyperactive peristalsis

Which teaching strategy is the nurses best choice for a person born before 1945? A.) Schedule group teaching sessions B.) Show video recordings on a television C.) Provide printed materials with diagrams D.) Downloaded information to an electronic device

C.) Provide printed materials with diagrams

Based on adult learning principles, which situation indicates that the patient is ready to learn about performing a subcutaneous injection at home? A.) The patient is requesting pain medication B.) The patient is falling asleep while watching the teaching DVD C.) The patient asks to practice before injecting himself with the needle D.) The patient is nervous and says he cannot do it as he picks up the syringe

C.) The patient asks to practice before injecting himself with the needle

Which situation would require the nurse to conduct a comprehensive assessment? A.) When performing resuscitation on a trauma patient B.) When assessing the progression of a patient's ascites and edema C.) When admitting a patient to the medical unit from the emergency department D.) When assessing a patient who has been receiving physical therapy after a stroke

C.) When admitting a patient to the medical unit from the emergency department

A nurse who is providing care for an older adult patient recognizes the need to maximize the patient's mobility during recovery from surgery. What accurately describes the best rationale for the nurse's actions? Continued activity prevents deconditioning. Pharmacokinetics are improved by patient mobility. Lack of stimulation contributes to the development of cognitive problems in older adults. Regularly scheduled physical rehabilitation provides an important sense of purpose for older patients.

Continued activity prevents deconditioning.

The patient was admitted via the emergency department with respiratory distress. What statement best described the initial assessment of the patient after he is transferred to the medical surgical unit? A.) An emergency assessment should be performed to ensure the patients safety B.) A focused assessment should be performed to be sure the ED did not overlook anything C.) A focused assessment should be performed to determine is new problems have arisen since transfer D.) A comprehensive assessment should be performed to determine the patients respiratory status and response to treatment

D.) A comprehensive assessment should be performed to determine the patients respiratory status and response to treatment

A 58 year old man is beign admitted tot he hospital for a thoracotomy and lung resection. Which assessment type would be best for this patient? A.) Focused B.) Emergency C.) Abbreviated D.) Comprehensive

D.) Comprehensive

The nurse is performing an assessment of a Chinese American patient wearing a jade bracelet on her left wrist. The nurse asks the patient about the significance of the bracelet and whether it can be removed before the scheduled diagnostic procedure. What behavior has the nurse demonstrated? A.) Diversity B.) Stereotyping C.) Ethnocentrism D.) Cultural competence

D.) Cultural competence

A patient is admitted with a fractured femur after a motor vehicle accident. While obtaining the health history, the nurse asks the patient about alcohol and substance use. What functional health pattern is being assessed? A.) Activity-exercise B.) Coping-stress tolerance C.) Self perception-self-concept D.) Health perception-health managements

D.) Health perception-health managements

What order of physical examination techniques would the nurse use when completing an abdominal assessment? A.) Inspection, palpation, percussion, and auscultation B.) Inspection, auscultation, palpation, and percussion C.) Auscultation, inspection, percussion, and palpation D.) Inspection, auscultation, percussion, and palpation

D.) Inspection, auscultation, percussion, and palpation

During the nurse's initial health assessment of an older adult patient, the nurse notes that the patient often makes self-deprecating statements, saying "I'm just a nuisance to my children now" and "It would be easier for everyone if I didn't have to be such a burden to them". The nurse would document a problem in which functional health pattern. A.) Value-belief B.) Cognitive-perceptual C.) Coping-stress tolerance D.) Self-perception-self-concept

D.) Self-perception-self-concept

A registered nurse has delegated IV medication administration to a licensed practical/vocational nurse. Which statement accurately describes delegation? A.) The RN must teach the LPN/VN how to administer the IV medications B.) Ultimate responsibility for administering the medication lies with the LPN/VN C.) The RN is responsible for observing the LPN/VN administer the IV medication D.) The RN is the one accountable for the quality of care that the patient receives

D.) The RN is the one accountable for the quality of care that the patient receives

Which action most clearly demonstrates a nursing focus rather than a medical focus? A.) The nurse recommends a change to the patients insulin sliding scale B.) The nurse interprets the results of the patients most recent glucose tolerance test C.) The nurse assesses the benefits of changing the patient to a new oral antihyperglycemic D.) The nurse assesses the effect that a new diagnosis of diabetes has on the patients lifestyle

D.) The nurse assesses the effect that a new diagnosis of diabetes has on the patients lifestyle

A patient with a long standing history of type one diabetes has brought several printouts from websites to the most recent visit with the primary care provider. Which statement would the nurse make to the patient regarding health information on the internet? A.) It is best to make sure that you avoid websites that are not associated with a government agency B.) You may have some good information there, but it is best to focus only on the information the doctor provides to you C.) There is a great deal of misinformation on the internet, so it is best to focus on printed material rather than electronic sources D.) There are websites that are credible and reliable, and I can give you information on making those decisions if you like

D.) There are websites that are credible and reliable, and I can give you information on making those decisions if you like

The nurse is interviewing an older adult patient. What is the priority nursing action during the interview process? Ensure all assistive devices are in place. Interview the patient and caregiver together. Perform the interview before administering analgesics. Move on to the next question if the patient does not respond quickly.

Ensure all assistive devices are in place.

A patient has a history of hypertension and type 1 diabetes. The patient exercises and eats a healthy diet. Which factors will most likely have a positive impact on biologic aging? (Select all that apply.) Exercise Diabetes Social support Good nutrition Coping resources

Exercise Social support Good nutrition Coping resources

An older adult patient admitted with an irregular heart rhythm has a lower blood level of medication than expected. Which common cause of medication errors made by older adults should the nurse suspect? Shorter drug half-life leading to medication overdose Decreased use of nonprescription over-the-counter drugs Improved tolerance of adverse effects of prescribed drugs Lack of financial resources to obtain prescribed medications

Lack of financial resources to obtain prescribed medications

A frail older adult with chronic heart failure is cared for in the home by her only child. Which problem is most indicated by the caregiver's failure to provide companionship or social stimulation? Neglect Sexual abuse Abandonment Violation of personal rights

Neglect

The nurse is administering medication to an older adult patient. Which consideration most related to aging would the nurse monitor when administering medications? The excretion of the medication The absorption of the medication How the medication is distributed The ability of the medication to metabolize

The ability of the medication to metabolize

The nurse is planning discharge for a frail older adult patient covered under Medicare Part A health insurance. Which medical supply would be covered if needed? Walker Dentures Eyeglasses Hearing aids

Walker

Which statement indicates the patient with asthma requires further teaching about self-care? a. "I use my corticosteroid inhaler every time I feel short of breath." b. "I get a flu shot every year and see my HCP if I have an upper respiratory tract infection." c. "I use my inhaler before I visit my aunt who has a cat, but I only visit for a few minutes because of my allergies." d. "I walk 30 minutes every day, but sometimes I have to use my bronchodilator inhaler before walking to prevent me from getting short of breath."

a. "I use my corticosteroid inhaler every time I feel short of breath."

When planning health care teaching to prevent or detect early head and neck cancer, which people would be the priority to target? (select all that apply) a. 65-years-old man who has used chewing tobacco most of his life b. 45-years-old rancher who uses chewing tobacco while driving his herds of cattle c. 21-years-old college student who drinks beer on weekends with his fraternity brothers d. 78-years-old woman who has been drinking liquor since her husband died 15 years ago e. 22-years-old woman who has been diagnosed with human papilloma virus of the cervix

a. 65-years-old man who has used chewing tobacco most of his life b. 45-years-old rancher who uses chewing tobacco while driving his herds of cattle d. 78-years-old woman who has been drinking liquor since her husband died 15 years ago e. 22-years-old woman who has been diagnosed with human papilloma virus of the cervix

Which patients are at most risk for pressure injuries? (select all that apply) a. A patient with right sided-paralysis and fecal incontinence b. An older adult who is alert and needs assistance to ambulate c. A young adult patient with paraplegia after a gunshot wound d. A morbidly obese patient who has an open abdominal wound e. An ambulatory patient who has occasional stress incontinence f. A young adult with a tibial fracture from a motor vehicle accident

a. A patient with right sided-paralysis and fecal incontinence c. A young adult patient with paraplegia after a gunshot wound d. A morbidly obese patient who has an open abdominal wound

Which treatment(s) would the nurse expect to implement in the management plan of an adult patient with cystic fibrosis? (select all that apply) a. Airway clearance techniques b. Bronchodilators and mucolytics c. Pancreatic enzyme replacement d. IV corticosteroids on a chronic basis e. Inhaled tobramycin to combat Pseudomonas infection

a. Airway clearance techniques b. Bronchodilators and mucolytics c. Pancreatic enzyme replacement e. Inhaled tobramycin to combat Pseudomonas infection

Which intervention would be part of the plan of care for a patient who has new vision loss? a. Allow the patient to express feelings of grief and anger. b. Have the AP perform all self-care activities for the patient. c. Address any family present first when discussing care concerns. d. Speak loudly and clearly, addressing the patient with each contact.

a. Allow the patient to express feelings of grief and anger.

Which finding indicate that a patient with asthma is developing status asthmaticus? (select all that apply) a. Anxiety and panic b. Positive sputum culture c. Unable to speak in complete sentences d. Chest x-ray shows hyperinflated lungs e. Lack of response to conventional treatment

a. Anxiety and panic c. Unable to speak in complete sentences e. Lack of response to conventional treatment

A patient with multiple gunshot wounds returns to the ICU from surgery. Vital signs are stable. They are making no spontaneous respiratory effort. Which mode of ventilation would be most appropriate? a. Assist control (AC) b. Pressure support (PS) c. Bi-level positive airway pressure (Bi-PAP) d. Synchronized intermittent mandatory ventilation (SIMV)

a. Assist control (AC)

The nurse teaches a patient with bronchiectasis that which problem would warrant a call to the clinic?" a. Blood clots in the sputum b. Sticky sputum on a hot day c. Producing large amounts of sputum daily d. Increased shortness of breath after eating a large meal

a. Blood clots in the sputum

The nurse is caring for a patient with diabetes in the ambulatory surgical unit who had a wound debridement. Which task is appropriate for the nurse to delegate to assistive personnel (AP)? a. Check the patient's vital signs. b. Assess the patient's pain level. c. Palpate the patient's pedal pulses. d. Monitor the patient's IV catheter site.

a. Check the patient's vital signs.

It is important for the nurse to assess for which manifestation(s) in a patient who has just undergone a total thyroidectomy? (select all that apply) a. Confusion b. Weight gain c. Depressed reflexes d. Circumoral numbness e. Positive Chvostek sign

a. Confusion d. Circumoral numbness e. Positive Chvostek sign

Nursing care of a patient with Stage 4 lung cancer would include a. Coordinating a referral to palliative care b. Limiting visitors to decrease infection risk c. NPO status and starting parenteral nutrition d. Avoiding talking about the cancer diagnosis

a. Coordinating a referral to palliative care

Common causes of genetic mutations include (select all that apply) a. DNA damage from toxins. b. DNA damage from UV radiation. c. inheritance of altered genes from father. d. inheritance of altered genes from mother. e. inheritance of somatic mutations from either parent.

a. DNA damage from toxins. b. DNA damage from UV radiation. c. inheritance of altered genes from father. d. inheritance of altered genes from mother.

Which action is the nurse's primary responsibility for the care of the patient undergoing surgery? a. Developing a patient-centered plan of nursing care b. Carrying out tasks related to surgical policies and procedure c. Ensuring that the patient has been assessed for safe administration of anesthesia d. Performing a preoperative history and physical assessment to identify patient needs

a. Developing a patient-centered plan of nursing care

Which statement most accurately describes cultural factors that may affect health? a. Diabetes and cancer rates differ by cultural/ethnic groups. b. There are limited ethnic variations in physiologic responses to drugs. c. Most patients find that religious rituals help them during times of illness. d. Silence after providing patient education means the patient understands the instructions.

a. Diabetes and cancer rates differ by cultural/ethnic groups.

What should be included in the discharge teaching for the patient who had cataract surgery? (Select all that apply.) a. Eye discomfort is often relieved with mild analgesics. b. A decline in visual acuity is common for the first month. c. Stay on bed rest and limit activity for the first few weeks. d. Notify the provider if an increase in redness or drainage occurs. e. Following activity restrictions is essential to reduce intraocular pressure.

a. Eye discomfort is often relieved with mild analgesics. d. Notify the provider if an increase in redness or drainage occurs. e. Following activity restrictions is essential to reduce intraocular pressure.

What interventions does the nurse anticipate for a patient with an aspirin overdose? (select all that apply.) a. Hemodialysis b. Corticosteroids c. Hyperbaric O2 d. Gastric lavage e. Activated charcoal

a. Hemodialysis d. Gastric lavage e. Activated charcoal

Which technique would be most appropriate for a patient with mild COPD to promote airway clearance? a. Huff coughing b. Postural drainage c. Pursed lip breathing d. High-frequency chest wall oscillation

a. Huff coughing

Which nursing action would be the highest priority when suctioning a patient with an oral ET tube or tracheostomy? a. Hyperoxygenate with 100% FIO2 before suctioning. b. Auscultate lung sounds after suctioning is completed. c. Instill 5 mL of normal saline into the tube before suctioning. d. Give antianxiety medications 30 minutes before suctioning.

a. Hyperoxygenate with 100% FIO2 before suctioning.

An 85-year-old patient has a score of 16 on the Braden Scale. What should the nurse include in the plan of care? a. Implementing a 1-hour turning schedule with skin assessment. b. Elevating the head of the bed 90 degrees when the patient is supine. c. Continuing with weekly skin assessments with no special precautions. d. Placing a silicone foam dressing on the patient's sacrum to prevent breakdown.

a. Implementing a 1-hour turning schedule with skin assessment.

Which factors does the nurse associate with insufficient sleep? (select all that apply) a. Increased body mass index b. Increased insulin resistance c. Impaired cognitive functioning d. Enhanced immune responsiveness e. Increased daytime body temperature

a. Increased body mass index b. Increased insulin resistance c. Impaired cognitive functioning

The nurse uses the knowledge of stress on the immune system through the understanding of which process? a. Infection risk increases with prolonged exposure to stress. b. The body's stress response does not affect cytokine production. c. Phagocytosis by neutrophils and monocytes are strengthened when illness occurs. d. Natural killer cells remain constant during both relaxation and stress periods.

a. Infection risk increases with prolonged exposure to stress.

A 59-year-old man scheduled for a herniorrhaphy in 2 days reports that he takes an anticoagulant agent daily. Which action would the nurse take? a. Inform the surgeon since the procedure may have to be rescheduled. b. Tell the patient to continue to take the drug up to the day before surgery. c. Ask the patient if he has any side effects from taking this drug supplement. d. Notify the anesthesia care provider since this drug may interfere with anesthetics.

a. Inform the surgeon since the procedure may have to be rescheduled.

An overweight patient with sleep apnea would like to avoid using a nasal CPAP device. Which recommendation would the nurse make to help the patient manage sleep apnea without using CPAP? a. Lose excess weight b. Take a nap each afternoon c. Eat a high-protein snack at bedtime d. Use mild sedatives or alcohol at bedtime

a. Lose excess weight

Which factors in positioning a patient for surgery increase the risk of patient injury? (select all that apply) a. Loss of pain perception b. Incorrect musculoskeletal alignment c. Vasoconstriction of the peripheral vessels d. Hypovolemia contributing to decreased perfusion e. Inability to sense pressure over bony prominences

a. Loss of pain perception b. Incorrect musculoskeletal alignment d. Hypovolemia contributing to decreased perfusion e. Inability to sense pressure over bony prominences

Management of a patient after a lung transplant includes which measures? (select all that apply) a. Mechanical ventilation in the early postoperative period b. Assisting with a lung biopsy if acute rejection is suspected c. IV fluid therapy accompanied by accurate intake and output d. Immunosuppressant therapy, which usually involves a 3 drug regimen e. Pulmonary clearance measures, including deep-breathing and coughing

a. Mechanical ventilation in the early postoperative period b. Assisting with a lung biopsy if acute rejection is suspected c. IV fluid therapy accompanied by accurate intake and output d. Immunosuppressant therapy, which usually involves a 3 drug regimen e. Pulmonary clearance measures, including deep-breathing and coughing

Which factors would the nurse include in discharge criteria for a Phase II patient? (select all that apply) a. Nausea and vomiting controlled. b. Ability to drive themselves home. c. No respiratory depression present. d. Written discharge instructions understood. e. Opioid pain medication given 45 minutes ago.

a. Nausea and vomiting controlled. c. No respiratory depression present. d. Written discharge instructions understood. e. Opioid pain medication given 45 minutes ago.

Which assessment finding concerns you most in a patient with pneumonia who is receiving noninvasive ventilation (Bi-PAP)? a. New onset of confusion to time and place b. Fine crackles on auscultation of affected lobe c. Patient asks to remove the mask for oral care d. HR: 98, RR: 16 bpm, BP: 110/60, SpO2: 93%

a. New onset of confusion to time and place

Which clinical manifestations are common in a patient who is actively dying? (select all that apply) a. Noisy secretions b. Increased sensation c. Mottling of hands and feet d. Cheyne-Stokes respirations e. Increased response to tactile stimuli

a. Noisy secretions c. Mottling of hands and feet d. Cheyne-Stokes respirations

After admitting a postoperative patient to the clinical unit, which assessment data require attention first? a. O2 saturation of 85% b. Respiratory rate of 13/min c. Temperature of 100.4°F (38°C) d. Blood pressure of 90/60 mm Hg

a. O2 saturation of 85%

Which factors place the patient at increased risk for severe COVID-19? (select all that apply) a. Obesity b. Thyroid disease c. Cigarette smoking d. Pernicious anemia e. Chronic kidney disease

a. Obesity c. Cigarette smoking e. Chronic kidney disease

A 70-kg postoperative patient has an average urine output of 25 mL/hr during the first 8 hours. Which interventions would the nurse prioritize? (Select all that apply.) a. Obtain a bladder ultrasound scan. b. Perform a straight catheterization. c. Continue to monitor this normal finding. d. Evaluate the patient's fluid volume status.

a. Obtain a bladder ultrasound scan. d. Evaluate the patient's fluid volume status.

Which patient behaviors would the nurse promote for healthy eyes? (select all that apply) a. Protective sunglasses when bicycling b. Taking part in a smoking cessation program c. Supplementing diet intake of vitamin C and beta-carotene d. Washing hands thoroughly before putting in or taking out contact lenses e. A woman avoiding pregnancy for 4 weeks after receiving MMR immunization

a. Protective sunglasses when bicycling b. Taking part in a smoking cessation program c. Supplementing diet intake of vitamin C and beta-carotene d. Washing hands thoroughly before putting in or taking out contact lenses

The nurse would place information about the patient's concern that his illness is threatening his job security in which functional health pattern? a. Role-relationship b. Cognitive-perceptual c. Coping-stress tolerance d. Health perception-health management

a. Role-relationship

Which findings indicate the patient is ready for weaning from mechanical ventilation? (select all that apply) a. Serum hemoglobin of 15 g/dL b. Respirations of 18 breaths/min c. Patient is alert and follow commands d. Chest x-ray shows large pleural effusion e. Mean arterial pressure (MAP) of 55 mm Hg f. ABGS: pH 7.38, PaCO2 37 mm Hg, 24 mEq/L, PaO2 94

a. Serum hemoglobin of 15 g/dL b. Respirations of 18 breaths/min c. Patient is alert and follow commands f. ABGS: pH 7.38, PaCO2 37 mm Hg, 24 mEq/L, PaO2 94

Which action would the nurse prioritize when limited time is available for patient teaching? a. Setting realistic goals that have high priority for the patient b. Referring the patient to a nurse educator for private practice c. Asking more experienced nurses how to teach faster and more efficiently d. Providing reading materials for the patient instead of discussing the information

a. Setting realistic goals that have high priority for the patient

A patient 1 day postoperative after abdominal surgery has incisional pain, 99.5°F temperature, slight redness at the incision margins, and 30 mL serosanguinous drainage in the Jackson-Pratt drain. Based on this assessment, what conclusion would the nurse make? a. The patient has a normal inflammatory response. b. The abdominal incision shows signs of an infection. c. The abdominal incision shows signs of impending dehiscence. d. The patient's health care provider must be notified about their condition.

a. The patient has a normal inflammatory response.

The nurse is preparing the patient for a diagnostic procedure to remove pleural fluid for analysis. The nurse would prepare the patient for which test? a. Thoracentesis b. Bronchoscopy c. Pulmonary angiography d. Sputum culture and sensitivity

a. Thoracentesis

A patient who recently started chemotherapy has uncontrollable nausea, diarrhea, muscle cramps, and dizziness. Which complication of cancer is this most likely caused by? a. Tumor lysis syndrome b. Third space syndrome c. Spinal cord compression d. Superior vena cava syndrome

a. Tumor lysis syndrome

The nurse is caring for a patient undergoing surgery for a knee replacement. Which factors are critical to the patient's safety during the procedure? (select all that apply) a. Universal protocol is followed. b. The ACP is an anesthesiologist. c. The patient has adequate health insurance. d. The patient's family is in the surgery waiting area. e. The patient's allergies are conveyed to the surgical team.

a. Universal protocol is followed. e. The patient's allergies are conveyed to the surgical team.

A nurse is leading a committee to improve LGBTQ+ inclusiveness on an inpatient medical unit. Which strategies are recommended for creating an inclusive LGBTQ+ environment? (select all that apply) a. Use a patient's preferred name and pronoun. b. Allow staff to wear a rainbow pin on their name badges. c. Assign a transgender patient to a private room, when possible. d. Make single stall public bathrooms to be all-gender bathrooms. e. Only allow a patient's immediate biologic family visiting rights.

a. Use a patient's preferred name and pronoun. b. Allow staff to wear a rainbow pin on their name badges. c. Assign a transgender patient to a private room, when possible. d. Make single stall public bathrooms to be all-gender bathrooms.

A 17-year-old patient with a leg fracture who is scheduled for surgery is an emancipated minor. She has a statement from the court for verification. Which action would the nurse take? a. Witness the patient signing the permit after the surgeon obtains consent. b. Call a parent or legal guardian to sign the permit since the patient is under 18. c. Notify the hospital attorney that an emancipated minor is consenting for surgery. d. Obtain verbal consent since written consent is not necessary for emancipated minors.

a. Witness the patient signing the permit after the surgeon obtains consent.

A student nurse asks the RN what can be measured by arterial blood gas (ABG). The RN tells the student that the ABG can measure (select all that apply) a. acid-base balance. b. bicarbonate (HCO3-). c. mixed venous O2 (SvO2). d. compliance and resistance. e. partial pressure of O2 (PaO2).

a. acid-base balance. b. bicarbonate (HCO3-). e. partial pressure of O2 (PaO2).

The nurse would suspect cocaine toxicity in the patient who is experiencing a. agitation, confusion, and seizures. b. diarrhea, nausea and vomiting, and confusion. c. blurred vision, constricted pupils, and paranoia. d. slow, shallow respirations; bradycardia; and hypotension.

a. agitation, confusion, and seizures.

The nurse's role in addressing the National Patient Safety Goals established by The Joint Commission includes (select all that apply) a. answering all patient monitoring alarms promptly. b. memorizing all the rules published by The Joint Commission. c. obtaining a correct list of the patient's medications on admission. d. encouraging patients to be actively involved in their health care. e. using side rails and alarm systems as necessary to prevent patient falls.

a. answering all patient monitoring alarms promptly. c. obtaining a correct list of the patient's medications on admission. e. using side rails and alarm systems as necessary to prevent patient falls.

An older man arrives in triage disoriented and dyspneic. His skin is hot and dry. His wife states that he was fine earlier today. The nurse's next priority would be to a. assess his vital signs. b. obtain a brief medical history from his wife. c. start supplemental O2 and have the provider see him. d. determine the kind of insurance he has before treating him.

a. assess his vital signs.

The nurse understands that a person who has chronic stress could be at higher risk for (select all that apply) a. asthma. b. osteoporosis. c. fibromyalgia. d. colds and influenza. e. high blood pressure.

a. asthma. d. colds and influenza. e. high blood pressure.

When caring for a patient with acute bronchitis, the nurse will prioritize interventions by a. auscultating lung sounds. b. encouraging fluid restriction. c. administering antibiotic therapy. d. teaching the patient to avoid cough suppressants.

a. auscultating lung sounds.

The key anatomic landmark that separates the upper respiratory tract from the lower respiratory tract is the a. carina. b. larynx. c. trachea. d. epiglottis.

a. carina.

In a spontaneously breathing patient, the nurse notes tidaling of the water level in the water-seal chamber of the chest tube drainage system. The nurse would a. continue to monitor the patient. b. check all connections for a leak in the system. c. raise the collection unit above the level of the heart. d. clamp the tubing at a distal point away from the patient.

a. continue to monitor the patient.

Defense mechanisms that help protect the lung from inhaled particles and microorganisms include the (select all that apply) a. cough reflex. b. mucociliary escalator. c. alveolar macrophages. d. reflex bronchoconstriction. e. alveolar capillary membrane.

a. cough reflex. b. mucociliary escalator. c. alveolar macrophages. d. reflex bronchoconstriction.

Surgery is used in cancer care to (select all that apply) a. diagnose cancer. b. cure or control cancer. c. provide supportive care. d. prevent spread of cancer. e. assist with rehabilitation after treatment.

a. diagnose cancer. b. cure or control cancer. c. provide supportive care. d. prevent spread of cancer. e. assist with rehabilitation after treatment.

Common age-related changes in the auditory system include (select all that apply) a. drier earwax. b. tinnitus in both ears. c. auditory nerve degeneration. d. atrophy of the tympanic membrane. e. greater ability to hear high-pitched sounds.

a. drier earwax. b. tinnitus in both ears. c. auditory nerve degeneration. d. atrophy of the tympanic membrane.

The most common cause of secondary immunodeficiencies is a. drugs. b. stress. c. malnutrition. d. human immunodeficiency virus.

a. drugs

The nurse would be alerted to possible anaphylactic shock after a patient has received IM penicillin by the development of a. edema and itching at the injection site. b. sneezing and itching of the nose and eyes. c. a wheal-and-flare reaction at the injection site. d. chest tightness and production of thick sputum.

a. edema and itching at the injection site.

The plan of care for the patient with chronic obstructive pulmonary disease (COPD) would include (select all that apply) a. exercise such as walking. b. high flow rate of O2 administration. c. low-dose oral corticosteroid therapy. d. use of monthly chest x-rays to monitor the progression of COPD. e. breathing exercises, such as pursed-lip breathing that focus on exhalation.

a. exercise such as walking.. c. low-dose oral corticosteroid therapy. e. breathing exercises, such as pursed-lip breathing that focus on exhalation.

The nursing care for a patient with hyponatremia and fluid volume excess includes a. fluid restriction. b. administration of hypotonic IV fluids. c. administration of a cation-exchange resin. d. placement of an indwelling urinary catheter.

a. fluid restriction.

Interventions to prevent health care-associated infections include (select all that apply) a. following hand-washing protocols. b. limiting visitors to persons over age 18. c. giving all patients prophylactic antibiotics. d. placing all patients on contact precautions. e. decontaminating equipment used for patient care.

a. following hand-washing protocols. e. decontaminating equipment used for patient care.

Care of the patient with an acute attack of Ménière disease includes (select all that apply) a. giving antiemetics as needed. b. implementing fall precautions. c. keeping the room dark and quiet. d. placing the patient on NPO status. e. ambulating in the hall independently.

a. giving antiemetics as needed. b. implementing fall precautions. c. keeping the room dark and quiet.

The patient who has a conductive hearing loss a. hears better in a noisy environment. b. hears sound but does not understand speech. c. often speaks loudly because their own voice seems low. d. has clearer sound with a hearing aid if the loss is less than 30 dB.

a. hears better in a noisy environment.

The lungs act as an acid-base buffer by a. increasing respiratory rate and depth when CO2 levels in the blood are high, reducing acid load. b. increasing respiratory rate and depth when CO2 levels in the blood are low, reducing base load. c. decreasing respiratory rate and depth when CO2 levels in the blood are high, reducing acid load. d. decreasing respiratory rate and depth when CO2 levels in the blood are low, increasing acid load.

a. increasing respiratory rate and depth when CO2 levels in the blood are high, reducing acid load.

Factors associated with an increase in reemerging infections include (select all that apply) a. international travel. b. poor immunization rates. c. poor sanitation standards. d. not completing a full course of antibiotics. e. correct use of personal protective equipment.

a. international travel. b. poor immunization rates. c. poor sanitation standards. d. not completing a full course of antibiotics.

The patient asks the nurse why they need viral load testing. The nurse responds that an undetectable HIV viral load (select all that apply) a. is the goal of HIV therapy. b. occurs with drug resistance. c. is a sign of disease progression. d. means that person is cured of HIV. e. means that someone is unable to sexually transmit HIV.

a. is the goal of HIV therapy e. means that someone is unable to sexually transmit HIV.

The nurse recognizes that a patient with newly diagnosed breast cancer is using an emotion-focused coping process when she a. joins a support group for women with breast cancer. b. considers the pros and cons of the various treatment options. c. delays treatment until her family can take a weekend trip together. d. tells the nurse that she has a good prognosis because the tumor is small.

a. joins a support group for women with breast cancer.

A woman has recently been diagnosed with early-stage uterine cancer. What can you do to promote effective coping strategies? (select all that apply) a. maintain the patient's hope. b. connect the patient to a support group. c. help the patient in setting realistic goals d. discuss replacement childcare for the patient's children. e. when patient begins to discuss her fears, change the topic to something uplifting.

a. maintain the patient's hope. b. connect the patient to a support group. c. help the patient in setting realistic goals

. While caring for a patient with alcohol withdrawal, the nurse should (select all that apply) a. monitor neurologic status on a routine basis. b. provide a quiet, nonstimulating, dimly lit environment. c. pad the side rails and place suction equipment at the bedside. d. orient the patient to the environment and person with each contact. e. give antiseizure drugs and sedatives to relieve withdrawal symptoms.

a. monitor neurologic status on a routine basis. c. pad the side rails and place suction equipment at the bedside. d. orient the patient to the environment and person with each contact. e. give antiseizure drugs and sedatives to relieve withdrawal symptoms.

Before injecting fluorescein for angiography, it is important for the nurse to (select all that apply) a. obtain an emesis basin. b. ask if the patient is fatigued. c. administer a topical anesthetic. d. inform patient that skin may turn yellow. e. assess for allergies to iodine-based contrast media.

a. obtain an emesis basin. d. inform patient that skin may turn yellow.

During administration of a hypertonic IV solution, the mechanism involved in equalizing the fluid concentration between ECF and the cells is (recognize) a. osmosis. b. diffusion. c. active transport. d. facilitated diffusion.

a. osmosis.

The trajectory of chronic illness includes (select all that apply) a. periods of crisis. b. episodes of exacerbations and stability. c. a gradual return to an acceptable way of life. d. a straight trajectory without overlapping phases. e. symptoms that can be controlled by proper treatment.

a. periods of crisis. b. episodes of exacerbations and stability. c. a gradual return to an acceptable way of life. e. symptoms that can be controlled by proper treatment.

A patient is seen at the clinic with fever, muscle aches, sore throat with yellowish exudate, and headache. The nurse anticipates that the interprofessional management will include (select all that apply) a. providing antipyretic for fever b. immediate treatment with antibiotics. c. a throat culture or rapid strep antigen test. d. supportive care, including cool, bland liquids. e. comprehensive history to determine possible cause

a. providing antipyretic for fever c. a throat culture or rapid strep antigen test. d. supportive care, including cool, bland liquids. e. comprehensive history to determine possible cause

The nurse would closely monitor patients exposed to a chlorine leak from a local factory for a. pulmonary edema. b. anaphylactic shock. c. respiratory acidosis. d. acute tubular necrosis.

a. pulmonary edema.

An important nursing responsibility related to pain is to (select all that apply) a. reassess the effect of analgesia. b. investigate potential causes of pain. c. trust the subjective pain report of the patient. d. perform pain assessments on nonverbal patients. e. withhold pain medication for patients with addiction.

a. reassess the effect of analgesia. b. investigate potential causes of pain. c. trust the subjective pain report of the patient. d. perform pain assessments on nonverbal patients.

If a person tests positive for a genetic mutation, it means (select all that apply) a. the laboratory found an alteration in a gene. b. the person is predisposed to develop a genetic disease. c. there is the chance other family members may be at risk. d. the person will definitely develop the disease at some point. e. the person should not have any children or any more children.

a. the laboratory found an alteration in a gene. c. there is the chance other family members may be at risk.

Always assess the patient with an eye problem for a. visual acuity. b. pupillary reactions. c. intraocular pressure. d. confrontation visual fields.

a. visual acuity.

An older adult is admitted to the medical unit with GI bleeding. Assessment findings that occur with fluid volume deficit include (select all that apply) a. weight loss. b. dry oral mucosa. c. full bounding pulse. d. engorged neck veins. e. orthostatic f. hypotension. g. increased central venous pressure.

a. weight loss. b. dry oral mucosa. e. orthostatic

A nurse is caring for a young man admitted with pneumonia. The patient discloses that he identifies as a gay male. Which statement by the nurse demonstrates an understanding of health disparities and the LGBTQ+ community? a. "Have you been vaccinated for measles?" b. "When was the last time you were tested for HIV?" c. "Do you need a referral to a hormone therapy specialist?" d. "When was the last time you received colon cancer screening?"

b. "When was the last time you were tested for HIV?"

Which patient has the greatest risk for delayed wound healing? a. A 65-year-old woman with stress incontinence b. A 52-year-old obese woman with type 2 diabetes c. A 78-year-old man who has a history of hypertension d. A 30-year-old man who drinks 2 alcoholic beverages per day

b. A 52-year-old obese woman with type 2 diabetes

Which situation would require the nurse to obtain a focused assessment? (select all that apply) a. A patient denies a current health problem. b. A patient reports a new symptom during rounds. c. A previously identified problem needs reassessment. d. A baseline health maintenance examination is needed. e. A patient with an emergent problem needs immediate care.

b. A patient reports a new symptom during rounds. c. A previously identified problem needs reassessment.

In a person having an acute rejection of a transplanted kidney, what would help the nurse understand the course of events? (select all that apply) a. A new transplant should be considered. b. Acute rejection can be treated with mycophenolate. c. Repeated episodes of acute rejection can lead to chronic rejection. d. Corticosteroids are the most successful drugs used to treat acute rejection. e. Acute rejection is common after a transplant and can be treated with drug therapy.

b. Acute rejection can be treated with mycophenolate. c. Repeated episodes of acute rejection can lead to chronic rejection. e. Acute rejection is common after a transplant and can be treated with drug therapy.

A 19 year-old patient arrives at the clinic with a sore throat and fever of 102.4°F (39.1°C). Assessment findings include enlarged anterior cervical lymph nodes and exudate at the back of the throat. What is the priority nursing action? a. Administer morphine 1 mg IV for pain b. Administer acetaminophen for pain and fever c. Increase hydration by providing oral and IV fluids d. Document the assessment findings in the nursing notes.

b. Administer acetaminophen for pain and fever

The patient with a right-side pleural effusion has stable vital signs and O2 at 6 L/min via nasal cannula. A right-side chest tube is attached to straight drainage. Which actions would the nurse include in the plan of care? (select all that apply) a. Placing the patient on NPO status b. Administering analgesia as ordered c. Maintaining high-Fowler's position d. Encouraging deep breathing and coughing e. Monitoring color and amount of chest tube drainage

b. Administering analgesia as ordered c. Maintaining high-Fowler's position d. Encouraging deep breathing and coughing e. Monitoring color and amount of chest tube drainage

When caring for a patient with a lung abscess, what is the nurse's priority intervention? a. Postural drainage b. Antibiotic administration c. Obtaining a sputum specimen d. Asking the patient about a family history of lung cancer

b. Antibiotic administration

The patient reports that she has noticed a skin reaction when wearing disposable gloves. Which action would the nurse take? a. Notify the surgeon so that the surgery can be canceled. b. Ask further questions to assess for a possible latex allergy. c. Notify the OR staff at once so they can use latex-free supplies. d. No action is needed because the patient's reaction has no bearing on surgery.

b. Ask further questions to assess for a possible latex allergy.

Which action is aligned with the 4M model of an age-friendly health system? a. Silencing a bed alarm so the patient can sleep at night b. Assessing if the patient needs a mobility device, such as a walker c. Asking for haloperidol for a patient with dementia who is pulling on their IV d. Telling the patient that they need to eat their dinner in order to avoid a feeding tube

b. Assessing if the patient needs a mobility device, such as a walker

What features of cancer cells distinguish them from normal cells? (select all that apply) a. Cancer cells proliferate faster. b. Cancer cells lack contact inhibition. c. Cancer cells regain a fetal appearance and function. d. Cancer cells return to a previous undifferentiated state. e. Proliferation occurs when there is a need for more cells.

b. Cancer cells lack contact inhibition. c. Cancer cells regain a fetal appearance and function. d. Cancer cells return to a previous undifferentiated state.

A patient admitted for scheduled surgery has a positive brief screening test for alcohol use disorder. Which initial action is most appropriate? a. Notify the health care provider. b. Complete a detailed alcohol use assessment. c. Initiate a referral to a specialty treatment center. d. Provide patient teaching on postoperative health risks.

b. Complete a detailed alcohol use assessment.

Which patient characteristic enhances learning outcomes? a. Feeling moderate anxiety b. Developing high self-efficacy c. Laughing about the current health problem d. Being in the precontemplation stage of change

b. Developing high self-efficacy

Which factors are part of the social determinants of a patient's health? (select all that apply.) a. Genetics b. Economic status c. Family history of disease d. Social and physical environment e. Type and quality of health care received

b. Economic status d. Social and physical environment e. Type and quality of health care received

What is the first step in developing cultural competence? a. Create opportunities to interact with a variety of cultural groups. b. Examine your own cultural background, values, and beliefs about health and health care. c. Learn about a multitude of folk medicines and herbal substances that different cultures use for self-care. d. Learn assessment skills for different cultural groups, including cultural beliefs and practices and physical assessments.

b. Examine your own cultural background, values, and beliefs about health and health care.

A 37-year-old patient is concerned they have asthma. Of the symptoms they describe to you, which ones suggest asthma or risk factors for asthma? (select all that apply) a. Prolonged inhalation b. Gastric reflux or heartburn c. Cough worse at night or early in the morning d. History of allergic rhinitis or chronic sinusitis e. Chest pain and syncope after exercising with a stationary bicycle for 5 minutes

b. Gastric reflux or heartburn c. Cough worse at night or early in the morning d. History of allergic rhinitis or chronic sinusitis

A patient who takes metformin 500 mg every morning for control of type 2 diabetes asks if she should take her medication the day of surgery. Which recommendation would the nurse make? a. Skip her medication the day of surgery. b. Get instructions from the surgeon about medication adjustments. c. Take her usual morning dose at bedtime the night before surgery. d. Take her medication as usual with a sip of water in the morning.

b. Get instructions from the surgeon about medication adjustments.

A patient in the unit has a 103.7°F temperature. Which intervention would be most effective in restoring normal body temperature? a. Using a cooling blanket while the patient is febrile b. Giving antipyretics on an around-the-clock schedule c. Providing increased fluids and have the AP give sponge baths d. Giving prescribed antibiotics and placing warm blankets for comfort

b. Giving antipyretics on an around-the-clock schedule

Which statements are true about rapid eye movement (REM) sleep? (select all that apply) a. The EEG pattern is quiescent. b. Muscle tone is greatly reduced. c. It occurs only once in the night. d. It is physiologically similar to NREM sleep. e. The most vivid dreaming occurs during this phase.

b. Muscle tone is greatly reduced. e. The most vivid dreaming occurs during this phase.

The nurse provides comprehensive pain management based on which principles? (select all that apply) a. Pain does not have an emotional component. b. Pain consists of an unpleasant sensory experience. c. Families and caregivers do not play a role in pain response. d. Self-report is one of the most valid means of pain assessment. e. Observed behaviors are used to assess pain in nonverbal patients.

b. Pain consists of an unpleasant sensory experience. d. Self-report is one of the most valid means of pain assessment. e. Observed behaviors are used to assess pain in nonverbal patients.

Which patient(s) have the greatest risk for aspiration pneumonia? (select all that apply) a. Patient who had thoracic surgery b. Patient with acute opioid overdose c. Patient who had a myocardial infarction d. Patient who is receiving nasogastric enteral feeding e. Patient who has a traumatic brain injury from blunt trauma

b. Patient with acute opioid overdose d. Patient who is receiving nasogastric enteral feeding e. Patient who has a traumatic brain injury from blunt trauma

Which activities might the nurse perform in the role of a scrub nurse during surgery? (select all that apply) a. Checking electrical equipment b. Preparing the instrument table c. Assisting with draping the patient d. Passing instruments to the surgeon and assistants e. Documenting activities occurring in the operating room

b. Preparing the instrument table c. Assisting with draping the patient d. Passing instruments to the surgeon and assistants

Which strategies would best aid the nurse communicate with a patient who has a hearing loss? (select all that apply) a. Overenunciate speech. b. Speak normally and slowly. c. Exaggerate facial expressions. d. Raise the voice to a higher pitch. e. Write out names or difficult words.

b. Speak normally and slowly. e. Write out names or difficult words.

What assessment technique should the nurse use to assess an adult patient's tympanic membrane? a. Have the patient tilt the head toward the nurse. b. Stabilize the otoscope with your fingers on the patient's cheek. c. Pull the auricle down and back to straighten the auditory canal. d. Use a speculum slightly larger than the size of the patient's ear canal.

b. Stabilize the otoscope with your fingers on the patient's cheek.

A 77-year-old man with emphysema is receiving palliative care along with pulmonary management. Which statements provide accurate information for planning this patient's care? (select all that apply) a. This patient does not need palliative care. b. The pulmonologist can provide primary palliative care. c. Palliative care can help improve coping and quality of life. d. Palliative care can help with symptom burden, such as dyspnea. e. The patient is at end of life since palliative care is only for those who are dying.

b. The pulmonologist can provide primary palliative care. c. Palliative care can help improve coping and quality of life. d. Palliative care can help with symptom burden, such as dyspnea.

A surgical unit's quality improvement committee notes the number of new catheter-associated urinary tract infections (CAUTIs) increased over the past 6 months. The nurse understands that this means: a. There is CAUTI pandemic on the unit. b. There is a need to review unit practices. c. Droplet precautions are needed to prevent CAUTIs. d. The prevalence of antibiotic resistant CAUTI infections is decreasing.

b. There is a need to review unit practices.

The nurse assessing a patient with a chronic leg wound finds redness and edema. The patient reports pain at the wound site. What would the nurse expect to be ordered to assess the patient's systemic response? a. Serum protein analysis b. WBC count and differential c. Punch biopsy of the center of the wound d. Culture and sensitivity of the wound

b. WBC count and differential

While caring for a patient with a history of narcolepsy with cataplexy, which activity can the nurse delegate to the assistive personnel (AP)? a. Teaching about the timing of medications b. Walking the patient to and from the bathroom c. Developing a plan of care with a family member d. Planning a diet that avoids caffeine-containing foods

b. Walking the patient to and from the bathroom

When admitting a patient, the nurse must assess the patient for substance use based on the knowledge that long-term use of addictive substances leads to a. the development of coexisting psychiatric illnesses. b. a higher risk for complications from underlying health problems. c. potentiation of effects of similar drugs taken when the person is drug-free. d. increased availability of dopamine, resulting in decreased sleep requirements.

b. a higher risk for complications from underlying health problems.

A nurse believes that all patients recovering from hip replacement surgery should have the same amount of pain. This statement reflects a. a belief that results in effective pain management. b. a lack of knowledge about factors affecting pain, leading to poor pain management. c. a belief that will not have any effect on the type of care provided to patients in pain. d. an accurate statement about pain mechanisms and the expected goals of pain therapy.

b. a lack of knowledge about factors affecting pain, leading to poor pain management.

A chemical explosion occurs at a nearby industrial site. First responders report that victims are decontaminated at the scene, and about 125 workers will need medical evaluation and care. The first action of the nurse receiving this report should be to a. issue a code blue alert. b. activate the hospital's emergency response plan. c. notify the Federal Emergency Management Agency (FEMA). d. arrange for the American Red Cross to provide aid to victims.

b. activate the hospital's emergency response plan.

A patient has a core temperature of 90°F (32.2°C). The most appropriate rewarming technique would be a. passive rewarming with warm blankets. b. active internal rewarming using warmed IV fluids. c. passive rewarming using air-filled warming blankets. d. active external rewarming by submersing in a warm bath.

b. active internal rewarming using warmed IV fluids.

To detect early signs or symptoms of inadequate oxygenation, the nurse would examine the patient for a. dyspnea and hypotension. b. apprehension and restlessness. c. cyanosis and cool, clammy skin. d. increased urine output and diaphoresis.

b. apprehension and restlessness.

A nurse is discharging an older adult patient who is homeless. Which actions demonstrate the nurse's understanding of the needs of this population? (select all that apply) a. instructs the patient to check his blood pressure daily b. asks the patient if they have a social worker or case manager c. inquires if the patient has concerns about staying in the local shelter d. asks the physician if enoxaparin can be changed to an oral anticoagulant e. informs the patient that the hospital will call with his culture test results next week

b. asks the patient if they have a social worker or case manager c. inquires if the patient has concerns about staying in the local shelter d. asks the physician if enoxaparin can be changed to an oral anticoagulant

A patient with an epidural catheter reports increased pain in their lower extremities. The best action by the nurse caring for this patient is to a. administer naloxone. b. assess the catheter for correct placement. c. ask the physician for an IV bolus of morphine. d. encourage the patient to take try a visualization technique.

b. assess the catheter for correct placement.

In a patient with a hemorrhage in the posterior cavity of the eye, the nurse knows that blood is accumulating a. in the aqueous humor. b. between the lens and retina. c. between the cornea and lens. d. in the space between the iris and lens.

b. between the lens and retina.

The nurse expects the long-term treatment of a patient with hyperphosphatemia from renal failure will include a. fluid restriction. b. calcium supplements. c. magnesium supplements. d. increased intake of dairy products.

b. calcium supplements.

Teach the patient who is newly fitted with bilateral hearing aids to (select all that apply) a. replace the batteries monthly. b. clean the ear molds weekly or as needed. c. clean ears with cotton-tipped applicators daily. d. disconnect or remove the batteries when not in use. e. initially restrict usage to quiet listening in the home.

b. clean the ear molds weekly or as needed. d. disconnect or remove the batteries when not in use. e. initially restrict usage to quiet listening in the home.

In identifying patients at the greatest risk for health disparities, the nurse would note that a. patients who live in urban areas have readily available access to health care services. b. cultural differences may exist in patients' ability to communicate with the health care provider. c. a patient receiving care from a health care provider of a different culture will have decreased quality of care. d. persons who immigrate to the United States are less likely to have health problems once they establish residency.

b. cultural differences may exist in patients' ability to communicate with the health care provider.

1. An example of a nursing activity that best reflects the American Nurses Association's definition of nursing is a. treating dysrhythmias that occur in a patient in the coronary care unit. b. diagnosing a patient with a feeding tube as being at risk for aspiration. c. setting up protocols for treating patients in the emergency department. d. offering antianxiety drugs to a patient with a disturbed sleep pattern.

b. diagnosing a patient with a feeding tube as being at risk for aspiration.

Principles of radiation treatment include (select all that apply) a. brachytherapy is delivered by an external beam. b. doses of radiation are expressed in units called gray or centigray. c. low energy beam radiation is ineffective on superficial skin lesions. d. the total dose for a radiation treatment is divided into daily fractions. e. radiosensitivity is the unresponsiveness of cells to the radiation treatment.

b. doses of radiation are expressed in units called gray or centigray. d. the total dose for a radiation treatment is divided into daily fractions.

When auscultating the chest of an older patient in mild respiratory distress, it is best to a. begin listening at the apices. b. dyspnea during rest or exercise. c. begin listening on the anterior chest. d. ask the patient to breathe through the nose with the mouth closed.

b. dyspnea during rest or exercise.

When assessing subjective data related to the respiratory health of a patient with emphysema, the nurse would ask about (select all that apply) a. date of last chest x-ray. b. dyspnea during rest or exercise. c. pulmonary function test results. d. ability to sleep through the entire night. e. prescription and over-the-counter medication.

b. dyspnea during rest or exercise. d. ability to sleep through the entire night. e. prescription and over-the-counter medication.

Expected effects of masculinizing hormone therapy include a. iron-deficiency anemia b. increased body and facial hair c. elevated thyroid stimulating hormone d. increased risk of pulmonary embolism and stroke

b. increased body and facial hair

After a left lower lobe lobectomy, an appropriate nursing measure is to a. position the patient prone every 2 hours. b. monitor the chest tube drainage and functioning. c. auscultate lung sounds frequently in the lower left lobe. d. administer IV fluid boluses to maintain blood pressure.

b. monitor the chest tube drainage and functioning.

Determination of whether an event is a stressor is based on a person's a. tolerance b. perception c. adaptation d. stubbornness

b. perception

The nurse teaches a patient scheduled for an electronystagmography that the test involves a. measuring ear drum movement in response to pressure. b. recording eye movements associated with ear irrigation. c. placing an electrode on the eardrum and assessing for dizziness. d. wearing headphones and determining which sounds can be heard.

b. recording eye movements associated with ear irrigation.

Presbyopia occurs in older people because a. the eyeball elongates. b. the lens becomes inflexible. c. the corneal curvature becomes irregular. d. light rays are focusing in front of the retina.

b. the lens becomes inflexible.

When communicating with a patient who speaks a language that the nurse does not understand, it is important to first attempt to a. have a family member interpret. b. use a trained medical interpreter. c. use specific medical terminology so there will be no mistakes. d. focus on the translation rather than nonverbal communication.

b. use a trained medical interpreter.

A basic principle to consider when planning treatment for HIV is a. stimulating the immune system to increase viral load. b. using a combination of drugs from more than one class. c. suppressing the replication of virus by decreasing the CD4+ count. d. encouraging patients to use natural supplements, such as St. John's Wort.

b. using a combination of drugs from more than one class.

As part of the nursing process, cultural assessment is best accomplished by a. judging the patient's cultural values based on observations. b. using a cultural assessment guide as part of the nursing process. c. seeking guidance from a nurse from the patient's cultural background. d. relying on the nurse's previous experience with patients from that cultural group.

b. using a cultural assessment guide as part of the nursing process.

When planning care for a patient at high risk for pulmonary embolism, the nurse prioritizes a. maintaining the patient on strict bed rest. b. using intermittent pneumatic compression devices. c. encouraging the patient to cough and deep breathe. d. encouraging a fluid intake of 2000 mL per 8-hour shift.

b. using intermittent pneumatic compression devices.

A patient reports tinnitus and balance problems. The medication that may be responsible is a. digoxin. b. warfarin. c. furosemide. d. acetaminophen.

b. warfarin.

Appropriate discharge teaching for the patient with a permanent tracheostomy after a total laryngectomy would include (select all that apply) a. encouraging regular exercise such as swimming. b. washing around the stoma daily with a moist washcloth. c. emphasizing the importance of regular follow-up appointments d. providing pictures and "hands-on" instruction for tracheostomy care. e. having a list of emergency contact numbers and where to obtain supplies.

b. washing around the stoma daily with a moist washcloth. c. emphasizing the importance of regular follow-up appointments d. providing pictures and "hands-on" instruction for tracheostomy care. e. having a list of emergency contact numbers and where to obtain supplies.

Appropriate discharge teaching for the patient with a permanent tracheostomy after a total laryngectomy for cancer would include (select all that apply) a. encouraging regular exercise such as swimming. b. washing around the stoma daily with a moist washcloth. c. encouraging participation in postlaryngectomy support group. d. providing pictures and "hands-on" instruction for tracheostomy care. e. teaching how to hold breath and trying to gag to promote swallowing reflex.

b. washing around the stoma daily with a moist washcloth. c. encouraging participation in postlaryngectomy support group. d. providing pictures and "hands-on" instruction for tracheostomy care.

The nurse explains to a patient with advanced prostate cancer about the differences between hospice and palliative care. Which patient statement indicates teaching was effective? a. "Hospice care is not available if I am in the hospital." b. "Palliative care provides better methods of pain control." c. "Hospice care can help me stay home and die naturally." d. "Palliative care does not include any advance directives."

c. "Hospice care can help me stay home and die naturally."

The nurse, who is teaching a patient how to manage chemotherapy-induced diarrhea, determines that teaching is successful when the patient states a. "I will only drink two glasses of liquid a day." b. "I should eat more fresh fruits and vegetables." c. "I will decrease fried and fatty foods in my diet." d. "Increasing the fiber in my diet will bulk up my stools."

c. "I will decrease fried and fatty foods in my diet."

A patient says she was diagnosed with astigmatism. When she asks what that is, what is the best explanation the nurse can give to the patient? a. "It happens because the lens of the eye is absent." b. "People with astigmatism have abnormally long eyeballs." c. "The cornea of the eye is uneven or irregular with astigmatism." d. "Astigmatism occurs because the eye muscles weaken with age."

c. "The cornea of the eye is uneven or irregular with astigmatism."

A nurse is completing the admission assessment for an 88-year-old woman admitted with heart failure. The nurse asks about advance directives, and the patient does not know what this means. Which response would accurately describe advance directives? a. "These are forms that tell your doctor when you want to withdraw care." b. "These forms will tell your loved ones where your money goes after you die." c. "There are forms such as a living will where you detail your treatment preferences." d. "You cannot fill out any advance directives right now since you are in the hospital."

c. "There are forms such as a living will where you detail your treatment preferences."

Which patient is at greatest risk for developing hypermagnesemia? a. 83-year-old man with lung cancer and hypertension b. 65-year-old woman with hypertension taking β-adrenergic blockers c. 42-year-old woman with systemic lupus erythematosus and renal failure d. 50-year-old man with benign prostatic hyperplasia and a urinary tract infection

c. 42-year-old woman with systemic lupus erythematosus and renal failure

A patient with TB is admitted to the hospital and placed in a single patient room on airborne precautions. What should the nurse teach the patient? (select all that apply) a. No visitors will be allowed while in airborne isolation. b. Expect regular TB skin testing to evaluate for infection. c. Adherence to precautions includes coughing into a paper tissue. d. Take all medications for full length of time to prevent multidrug-resistant TB. e. Wear a standard isolation mask if leaving the airborne infection isolation room.

c. Adherence to precautions includes coughing into a paper tissue. d. Take all medications for full length of time to prevent multidrug-resistant TB. e. Wear a standard isolation mask if leaving the airborne infection isolation room.

The nurse taught a caregiver how to administer insulin. Which action would the nurse take to evaluate the caregiver's learning? a. Monitor the patient's glucose readings. b. Arrange for follow-up with a home care nurse. c. Ask the caregiver to "show back" the ability to administer insulin. d. Assess what the caregiver thought was effective about the teaching.

c. Ask the caregiver to "show back" the ability to administer insulin.

Which actions would the nurse prioritize when admitting a patient to the PACU? a. Assess the surgical site, noting presence and character of drainage. b. Assess the amount of urine output and the presence of bladder distention. c. Assess for airway patency and quality of respirations and obtain vital signs. d. Review results of intraoperative laboratory values and medications received.

c. Assess for airway patency and quality of respirations and obtain vital signs.

Which order should a nurse question in the plan of care for an older adult, immobile stroke patient with a pink, clean stage 3 pressure injury? a. Pack the wound with foam dressing. b. Turn and position the patient every hour. c. Clean the wound daily with a cytotoxic solution. d. Assess for pain and medicate before dressing change.

c. Clean the wound daily with a cytotoxic solution.

During a stressful circumstance that is unchangeable, which type of coping strategy is the most effective? a. Avoidance b. Coping flexibility c. Emotion-focused coping d. Problem-focused coping

c. Emotion-focused coping

An older woman arrives in the ED reporting severe pain in her right shoulder. The nurse notes her clothes are soiled with urine and feces. She tells the nurse that she lives with her son and that she "fell." She is tearful and asks you if she can be admitted. What possibility should the nurse consider? a. Dementia b. Possible cancer c. Family violence d. Orthostatic hypotension

c. Family violence

You are caring for a transgender woman who recently underwent vaginoplasty. What information should the nurse include as part of the patient's discharge instructions? a. You will need regular cervical cancer screening. b. Let me show you how to do testosterone injections. c. If you find the dilators difficult to use, contact your surgeon. d. You should anticipate your first menstrual cycle to begin soon.

c. If you find the dilators difficult to use, contact your surgeon.

Newborns are protected for the first 3 months of life from bacterial infections because of the maternal transmission of a. IgA. b. IgE. c. IgG. d. IgM.

c. IgG.

The nurse would expect which finding in a patient due to the physiologic effect of stress on the reticular formation? a. An episode of diarrhea while awaiting painful dressing changes b. Refusal to communicate with nurses while awaiting a cardiac catheterization c. Inability to sleep the night before beginning to self-administer insulin injections d. Increased blood pressure, decreased urine output, and hyperglycemia after a car accident

c. Inability to sleep the night before beginning to self-administer insulin injections

Why is IV induction for general anesthesia the method of choice for most patients? a. The patient is not intubated. b. The agents are nonexplosive. c. Induction is rapid and controlled. d. Emergence is longer but with fewer complications.

c. Induction is rapid and controlled.

You are caring for a patient with several traumatic injuries after a multiple-vehicle accident. Which assessment finding would lead you to suspect a flail chest? a. Chest-tube is draining bright red blood b. Tracheal deviation to the unaffected side c. Paradoxical chest movement during respiration d. Little to no movement of the involved chest wall

c. Paradoxical chest movement during respiration

Which rationale supports including caregivers in patient teaching? (select all that apply.) a. Caregivers provide all the care for patients after discharge. b. They might feel rejected if they are not included in the teaching. c. Patients have better outcomes when their caregivers are involved. d. The patient may be too ill or too stressed to fully understand the teaching. e. Caregivers are responsible for the overall management of the patient's care.

c. Patients have better outcomes when their caregivers are involved.

Which goal applies when planning to teach a middle-aged woman about ways to relieve symptoms of menopause? a. Prevent the development of future disease. b. Maintain the patient's current state of health. c. Provide information on possible treatment options. d. Change the patient's beliefs about herbal supplements.

c. Provide information on possible treatment options.

Which preoperative considerations would the nurse plan for the care of an older adult? (Select all that apply.) a. Using only large-print educational materials. b. Speaking louder for patients with hearing aids. c. Recognizing that sensory deficits may be present. d. Providing warm blankets to prevent hypothermia. e. Teaching important information early in the morning.

c. Recognizing that sensory deficits may be present. d. Providing warm blankets to prevent hypothermia.

In which situation would the nurse suspect elder mistreatment? a. Patient admitted with recurrent syncope b. Creatinine of 1.1 mg/dL and BUN of 10 mg/dL c. Sacral pressure injury on a patient who lives at home d. Patient with dementia who becomes more confused at night

c. Sacral pressure injury on a patient who lives at home

A nurse is caring for a patient who has a pressure injury that is treated with debridement, irrigations, and moist gauze dressings. How would the nurse expect healing to occur? a. Cell regeneration b. Tertiary intention c. Secondary intention d. Remodeling of tissues

c. Secondary intention

Which strategy reduces drowsiness on the job for night shift workers? a. Exercising before going to work b. Taking melatonin before working the night shift c. Sleeping for at least 2 hours immediately before work time d. Walking for 10 minutes every 4 hours during the night shift

c. Sleeping for at least 2 hours immediately before work time

An 82-year-old man is being cared for at home by his family. A pressure injury on his right buttock measures 1 × 2 × 0.8 cm, and pink subcutaneous tissue is completely visible on the wound bed. Which stage would the nurse document on the wound assessment form? a. Stage 1 b. Stage 2 c. Stage 3 d. Stage 4

c. Stage 3

The nurse should be alert for which manifestations in a patient receiving a loop diuretic? a. Restlessness and agitation b. Paresthesias and irritability c. Weak, irregular pulse and poor muscle tone d. Increased blood pressure and muscle spasms

c. Weak, irregular pulse and poor muscle tone

Demographic trends among older Americans in the United States suggests a. there are fewer people living past age 85. b. more frailty in persons between 65 and 75 years. c. a growth in racial and ethnically diverse populations. d. women having a decreased life expectancy when compared to men.

c. a growth in racial and ethnically diverse populations.

Ask patients using eyedrops to treat their glaucoma about a. use of corrective lenses. b. their usual sleep pattern. c. a history of heart or lung disease. d. sensitivity to opioids or depressants.

c. a history of heart or lung disease.

The nurse caring for a patient who identifies as bisexual understands that bisexuality refers to a. a person who was born female but uses testosterone to grow facial hair. b. a person born male who uses estrogen to achieve feminine characteristics. c. a person who is sexually or affectionately attracted to more than one gender. d. a person who expresses their gender by wearing both male and female clothing.

c. a person who is sexually or affectionately attracted to more than one gender.

To prevent fever and shivering during an infusion of rituximab (Rituxan), the nurse should premedicate the patient with a. aspirin. b. diazepam. c. acetaminophen. d. sodium bicarbonate.

c. acetaminophen.

The nurse can best determine adequate arterial oxygenation of the blood by assessing a. heart rate. b. hemoglobin level. c. arterial oxygen partial pressure. d. arterial carbon dioxide partial pressure.

c. arterial oxygen partial pressure.

The most appropriate nursing intervention for a patient who is being treated for an exacerbation of emphysema and is not interested in quitting smoking is to a. accept the patient's decision and not intervene until the patient expresses a desire to quit. b. realize that some smokers never quit, and trying to assist them increases the patient's frustration. c. ask the patient to identify the risks and benefits of quitting and what barriers to quitting are present. d. motivate the patient to quit by describing how continued smoking will worsen the breathing problems.

c. ask the patient to identify the risks and benefits of quitting and what barriers to quitting are present.

The nurse is assessing a postoperative patient for the presence of pain. The best approach to assess this patient's pain is to a. take the patient's vital signs. b. observe for nonverbal pain behaviors. c. ask the patient to rate their pain on a 0 to 10 scale. d. question the caregiver as to their belief if the patient is in pain.

c. ask the patient to rate their pain on a 0 to 10 scale.

A patient is seen in the clinic for a nosebleed, which is controlled by placing anterior nasal packing. During discharge teaching, the nurse teaches the patient to a. use aspirin for pain relief. b. remove the packing later that day. c. avoid vigorous nose blowing and strenuous activity. d. insert more packing into the nose if re-bleeding occurs.

c. avoid vigorous nose blowing and strenuous activity.

A patient is receiving a patient-controlled analgesia (PCA) infusion after spinal surgery. She is sleeping soundly, awakens when the nurse speaks to her in a normal tone of voice, and reports her pain as "mild and tolerable." Her respirations are 8 breaths/min. The most appropriate nursing action in this situation is to a. stop the PCA infusion. b. obtain an oxygen saturation level. c. continue to closely monitor the patient. d. administer naloxone and contact the provider.

c. continue to closely monitor the patient.

The major advantage of a Venturi mask is that it can a. deliver up to 80% O2. b. provide continuous 100% humidity. c. deliver a precise concentration of O2. d. be used while a patient eats and sleeps.

c. deliver a precise concentration of O2.

A patient is undergoing plasmapheresis for treatment of systemic lupus erythematosus. The nurse explains that plasmapheresis is used in treatment to a. remove T lymphocytes in her blood that are producing antinuclear antibodies. b. remove normal particles in her blood that are being damaged by autoantibodies. c. exchange her plasma that contains antinuclear antibodies with a substitute fluid. d. replace viral-damaged cellular components of her blood with replacement whole blood.

c. exchange her plasma that contains antinuclear antibodies with a substitute fluid.

2. A nurse working on the medical-surgical unit at an urban hospital would like to become certified in medical-surgical nursing. The nurse knows that this process would most likely require a. a bachelor's degree in nursing. b. formal education in advanced nursing practice. c. experience for a specific period in medical-surgical nursing. d. membership in a medical-surgical nursing specialty organization.

c. experience for a specific period in medical-surgical nursing.

During the respiratory assessment of an older adult, the nurse would expect to find (select all that apply) a. a vigorous reflex cough. b. increased chest expansion. c. increased residual volume. d. decreased lung sounds at base of lungs. e. increased anteroposterior (AP) chest diameter.

c. increased residual volume. d. decreased lung sounds at base of lungs. e. increased anteroposterior (AP) chest diameter.

A patient with pancreatitis reports abdominal pain. The nurse knows that this type of pain a. is described as an electrical shock. b. arises from damage to peripheral nerves. c. is associated with visceral nociceptive pain. d. is best treated with adjuvant therapies, such as gabapentin.

c. is associated with visceral nociceptive pain.

The typical fluid replacement for the patient with a fluid volume deficit is a. dextran. b. 0.45% saline. c. lactated Ringer's solution. d. 5% dextrose in 0.45% saline.

c. lactated Ringer's solution.

A normal finding the nurse would expect when assessing hearing would be a. absent cone of light. b. bluish purple tympanic membrane. c. midline tone heard equally in both ears. d. fluid level at hairline in the tympanic membrane.

c. midline tone heard equally in both ears.

Substance use problems in older adults are usually related to a. use of drugs and alcohol as a social activity. b. continuing the use of illegal drugs started during middle age. c. misuse of prescribed and over-the-counter medications and alcohol. d. a pattern of binge drinking for weeks or months with periods of sobriety.

c. misuse of prescribed and over-the-counter medications and alcohol.

Nursing management of a patient with an oral ET tube would include a. maintaining ET tube cuff pressure at 35 to 40 cm H2O. b. routine suctioning of the ET tube at least every 2 hours. c. observing the patient for spontaneous respiratory effort and work of breathing. d. preventing ET tube dislodgment by limiting mouth care to lubrication of the lips.

c. observing the patient for spontaneous respiratory effort and work of breathing.

Opportunistic diseases in HIV infection a. are usually benign. b. are slow to develop and progress. c. occur in the presence of immunosuppression. d. are curable with appropriate drug interventions.

c. occur in the presence of immunosuppression.

A father who has an X-linked recessive disorder and a wife with a normal genotype will a. pass the carrier state to all his children. b. pass the carrier state to his male children. c. pass the carrier state to his female children. d. not pass on the genetic mutation to any of his children.

c. pass the carrier state to his female children.

When working with a patient who has suspected tuberculosis, the nurse would a. wear a cloth mask. b. only use standard precautions. c. place the patient on airborne precautions. d. wear a new gown each time they enter the room.

c. place the patient on airborne precautions.

The immune system monitors and regulates proliferation of cancer cells through (select all that apply) a. suppressing cytotoxic T cells. b. deactivating natural killer cells. c. producing antibodies that bind to tumor cells. d. surveillance of cells with tumor-associated antigens. e. sensitizing natural killer cells before they lyse tumor cells.

c. producing antibodies that bind to tumor cells. d. surveillance of cells with tumor-associated antigens.

Nursing interventions directed at health promotion in the older adult are mainly focused on a. performing IADLs. b. symptom management. c. reducing risk for illness or injury. d. assessing if the patient has an advanced directive.

c. reducing risk for illness or injury.

The nurse monitors the patient with positive pressure mechanical ventilation for a. paralytic ileus because pressure on the abdominal contents affects bowel motility. b. diuresis and sodium depletion because of increased release of atrial natriuretic peptide. c. signs of cardiovascular insufficiency because pressure in the chest impedes venous return. d. respiratory acidosis in a patient with COPD because of alveolar hyperventilation and increased PaO2 levels.

c. signs of cardiovascular insufficiency because pressure in the chest impedes venous return.

A patient is ordered cardiac rehabilitation following cardiac bypass surgery. The nurse recognizes this as a. primary prevention for atelectasis. b. secondary prevention for atherosclerosis. c. tertiary prevention to reduce the progression of heart disease. d. a recommended treatment to prevent deep vein thrombosis.

c. tertiary prevention to reduce the progression of heart disease.

The most effective method of administering a chemotherapy agent that is a vesicant is to a. give it orally. b. give it intraarterially. c. use a central venous access device. d. use the smallest gauge needle through a peripheral line.

c. use a central venous access device.

When using evidence-based practice, the nurse a. must use clinical practice guidelines developed by national health agencies. b. should use findings from randomized controlled trials to plan care for all patient problems. c. uses clinical decision making and judgment to decide what evidence is appropriate for a specific clinical situation. d. analyzes the relationship of nursing interventions to patient outcomes to discover evidence for patient interventions.

c. uses clinical decision making and judgment to decide what evidence is appropriate for a specific clinical situation.

In a patient with vertigo, the parts of the ear most likely involved are the (select all that apply) a. cochlea. b. ossicles. c. vestibule. d. semicircular canals. e. tympanic membrane.

c. vestibule. d. semicircular canals.

Which statement best describes sleep? a. A loosely organized state similar to coma b. A quiet state in which there is little brain activity c. A state in which a person has reduced sensitivity to pain d. A state lacking conscious awareness of the environment

d. A state lacking conscious awareness of the environment

Which medication would be appropriate to administer to a patient experiencing an acute asthma attack? a. IV theophylline b. Montelukast (Singulair) c. Inhaled hypertonic saline d. Albuterol (Proventil HFA)

d. Albuterol (Proventil HFA)

The nurse is assigned a 49-year-old patient with breast cancer who speaks only Cantonese. There are several family members at bedside. Which action would the nurse take first? a. Call a pastor or priest for the family to help them cope. b. Avoid using a translator for fear of offending the family. c. Ask for a different nurse to have this patient assignment. d. Assess the language preferences and beliefs of the patient and family.

d. Assess the language preferences and beliefs of the patient and family.

What is the most appropriate nursing intervention to help patients with HIV adhere to their treatment plan? a. Set up a drug pillbox for the patient every week. b. Give the patient a video and a brochure to view and read at home. c. Tell the patient that side effects of ART are bad but that they go away. d. Assess the patient's routines and find adherence cues that fit into their life circumstances.

d. Assess the patient's routines and find adherence cues that fit into their life circumstances.

Association between HLA antigens and diseases is most often found in what disease conditions? a. Cancers b. Infectious diseases c. Neurologic diseases d. Autoimmune disorders

d. Autoimmune disorders

Which respiratory assessment finding does the nurse interpret as abnormal? a. Inspiratory chest expansion of 1 inch b. Symmetric chest expansion and contraction c. Resonance (to percussion) over the lung bases d. Bronchial breath sounds in the lower lung fields

d. Bronchial breath sounds in the lower lung fields

What characterizes multifactorial genetic disorders? a. Often caused by single-gene alterations b. Genetic testing available for most disorders c. Many family members report having the disorder d. Caused by complex interactions of genetic and environmental factors

d. Caused by complex interactions of genetic and environmental factors

Which intervention would the nurse prioritize to aid a preoperative patient in coping with the fear of postoperative pain? a. Inform the patient that pain medication will be available. b. Teach the patient to use guided imagery to help manage pain. c. Describe the type of pain expected after the patient's surgery. d. Explain the pain management plan and the use of a pain rating scale.

d. Explain the pain management plan and the use of a pain rating scale.

A patient tells the nurse that he is ready to start a weight-loss program. Which nursing action would match this patient's readiness for change? a. Confirm that the patient is serious about losing weight. b. Assess the patient's awareness of his dietary behaviors. c. Insist that the patient consider an organized group weight-loss program. d. Focus on the patient's strengths to support his optimism that he can lose weight.

d. Focus on the patient's strengths to support his optimism that he can lose weight.

Which action would the nurse take when scrubbing at the scrub sink? a. Scrub from elbows to hands b. Scrub without mechanical friction c. Scrub for a minimum of 10 minutes d. Hold the hands higher than the elbows

d. Hold the hands higher than the elbows

What accurately describes rejection after transplantation? a. Hyperacute rejection can be treated with antimetabolites. b. Acute rejection can be treated with sirolimus or tacrolimus. c. Chronic rejection can be treated with tacrolimus or cyclosporine. d. Hyperacute reaction can be avoided if crossmatching is done before transplantation.

d. Hyperacute reaction can be avoided if crossmatching is done before transplantation.

What role does pharmacogenomics have in health care? a. It can assess individual variability to many drugs. b. Information can assess the effectiveness of a drug. c. It provides important assessment data for gene therapy. d. It can assess the variability of drug responses due to single genes.

d. It can assess the variability of drug responses due to single genes.

A patient is scheduled for surgery requiring general anesthesia at an ambulatory surgical center. The nurse asks him when he ate last. He replies that he had a light breakfast a couple of hours before coming to the surgery center. Which action would the nurse take? a. Tell the patient to come back tomorrow since he ate a meal. b. Have the patient void before giving any preoperative medication. c. Proceed with the preoperative checklist, including site identification. d. Notify the anesthesia care provider of when and what the patient last ate.

d. Notify the anesthesia care provider of when and what the patient last ate.

A patient is admitted to the PACU after major abdominal surgery. During the initial assessment the patient tells the nurse, "I think I am going to throw up." Which is the priority intervention? a. Increase the rate of the IV fluids. b. Give antiemetic medication as ordered. c. Obtain vital signs, including O2 saturation. d. Position patient in lateral recovery position.

d. Position patient in lateral recovery position.

A woman has been widowed for 2 years after her husband died from glioblastoma. The woman stopped working, does not leave the house often, has not removed her husband's items from the home, and cries daily about her loss. How would the nurse characterize this woman's grieving? a. Adaptive grief b. Disruptive grief c. Anticipatory grief d. Prolonged grief disorder

d. Prolonged grief disorder

A 35-year-old woman with sarcoma arrives to the clinic for chemotherapy. She reports severe pain at a level of 7 (0- to 10-point scale) in her right lower extremity. Which action would the nurse take before starting the chemotherapy? a. Auscultate for bowel sounds. b. Ask her about advance directives. c. Administer chemotherapy premedications. d. Review orders and give prescribed pain medications.

d. Review orders and give prescribed pain medications.

When examining the patient's eyes, which finding would be of most concern to the nurse? a. Intraocular pressure of 16 mm Hg b. Slightly yellowish cast of the sclera c. Outward turning of the lower lid margin d. Small, white nodule on the upper lid margin

d. Small, white nodule on the upper lid margin

A patient tells the nurse that she enjoys talking with others and sharing experiences but often falls asleep when reading. Which teaching strategy would the nurse plan for this patient? a. Formal lecture b. Journal writing c. Digital technology d. Small-group discussion

d. Small-group discussion

A patient with obesity (BMI 42.1 kg/m2) is scheduled for a laparoscopic cholecystectomy in an outpatient surgery setting. Which information would the nurse include in the plan of care? a. The patient will be in the hospital for several days. b. Surgery will involve removing a part of the liver. c. The setting is not appropriate for the planned procedure. d. Special equipment may be needed for the patient's care.

d. Special equipment may be needed for the patient's care.

A patient on home hospice tells the nurse, "I can't believe God would do this to me. I've always been healthy until this cancer. I don't understand!" Which state is the patient exhibiting? (select all that apply) a. Denial b. Loneliness c. Fear of death d. Spiritual distress e. Existential distress

d. Spiritual distress e. Existential distress

A patient receiving treatment for cancer is having problems maintaining their weight. What is the best option that the nurse can do for the patient? a. Counsel them to eat foods low in fiber. b. Recommend high-fat foods that are spicy. c. Contact the provider about a feeding tube. d. Suggest adding nutrition supplements with meals.

d. Suggest adding nutrition supplements with meals.

Which items would the nurse wear for proper attire in the semirestricted area of the surgery department? a. Street clothing b. Surgical attire and head cover c. Street clothing and shoe covers d. Surgical attire, head cover, shoe covers

d. Surgical attire, head cover, shoe covers

A patient on chemotherapy and radiation for head and neck cancer has a WBC count of 1.9 × 103/μL, hemoglobin of 10.8 g/dL, and a platelet count of 99 × 103/μL. Based on the CBC results, which are the most serious clinical findings? a. Anorexia and nausea b. Headache, mucositis, and constipation c. Fatigue and skin redness at site of radiation d. Temperature of 101.9°F, fatigue, and shortness of breath

d. Temperature of 101.9°F, fatigue, and shortness of breath

When teaching the patient with insomnia about sleep hygiene, which information would the nurse emphasize? a. The importance of daytime naps b. The need for long-term use of hypnotics c. Planning for exercise about 1 hour before bedtime d. The importance of avoiding caffeine 6 to 9 hours before bedtime

d. The importance of avoiding caffeine 6 to 9 hours before bedtime

What would the nurse include when teaching a patient how to use a metered-dose inhaler (MDI)? a. After activating the MDI, breathe in as quickly as you can. b. Estimate the amount of remaining medicine in the MDI by floating the canister in water. c. Disassemble the plastic canister from the inhaler and rinse both pieces under running water every week. d. To determine how long the canister will last, divide the total number of puffs in the canister by the puffs needed per day.

d. To determine how long the canister will last, divide the total number of puffs in the canister by the puffs needed per day.

A patient asks, "How does air get into my lungs?" The nurse bases their answer on knowledge that air moves into the lungs because of a. positive intrathoracic pressure. b. contraction of the accessory abdominal muscles. c. stimulation of the respiratory muscles by the chemoreceptors. d. a decrease in intrathoracic pressure from an increase in thoracic cavity size.

d. a decrease in intrathoracic pressure from an increase in thoracic cavity size.

An appropriate nursing intervention for a hospitalized patient who says she cannot cope with her illness is a. controlling the environment to prevent sensory overload and promote sleep. b. encouraging the patient's family to offer emotional support by frequent visiting. c. arranging for the patient to phone family and friends to maintain emotional bonds. d. asking the patient to describe previous stressful situations and how she managed to resolve them.

d. asking the patient to describe previous stressful situations and how she managed to resolve them.

Which action is a priority for a newly admitted patient? a. checking for pressure ulcers b. planning for post-discharge needs c. administering an influenza vaccine d. assessing the patient's mental status

d. assessing the patient's mental status

A patient with cancer who reports constant, moderate pain with short periods of severe pain during dressing changes. The best approach in managing this patient's pain is a. intermittent heat and cold therapy. b. acetaminophen only when the patient asks for it. c. as needed long-acting opioids and topical lidocaine. d. best treated by receiving a long-acting and a short-acting opioid.

d. best treated by receiving a long-acting and a short-acting opioid.

The nurse caring for a patient with HIV who has been on ART for many years plans care with the knowledge that the patient a. will inevitably develop drug resistance. b. is less likely to develop problems such as hyperlipidemia. c. is more likely to require treatment of opportunistic infections. d. can develop other chronic conditions at an earlier age than someone without HIV.

d. can develop other chronic conditions at an earlier age than someone without HIV

A patient with a low number of monocytes would have a decreased ability to a. stimulate the production of T and B lymphocytes. b. make antibodies after exposure to foreign substances. c. bind antigens and stimulate natural killer cell activation. d. capture antigens by phagocytosis and present them to lymphocytes.

d. capture antigens by phagocytosis and present them to lymphocytes.

A characteristic of the promotion stage of cancer development is a. tumor cells detach from the primary tumor. b. tumor cells are trapped in a sentinel lymph node. c. genetic mutations of cells initially occur in this stage. d. cellular alterations that occur in this stage are reversible.

d. cellular alterations that occur in this stage are reversible.

Forcing one's own cultural beliefs and practices on another person is an example of a. stereotyping. b. ethnocentrism. c. cultural relativity. d. cultural imposition.

d. cultural imposition.

Unrelieved pain is a. inevitable in persons in hospice. b. expected in a person with peripheral neuropathy. c. something to expect because of the opioid addiction epidemic. d. dangerous and can lead to many physical and psychologic complications.

d. dangerous and can lead to many physical and psychologic complications.

An important mechanism in promoting health equity is to a. discourage use of evidence-based practice guidelines. b. insist that patients adhere to established clinical guidelines. c. teach patients to use the Internet to find resources related to their health. d. engage in active listening and establish relationships with patients and families.

d. engage in active listening and establish relationships with patients and families.

A couple who recently had a son with hemophilia A is consulting with a nurse. They want to know if their next child will have hemophilia A. The nurse can tell the parents that if their child is a a. boy, he will have hemophilia A. b. boy, he will be a carrier of hemophilia A. c. girl, she will be a carrier of hemophilia A. d. girl, there is a 50% chance she will be a carrier of hemophilia A.

d. girl, there is a 50% chance she will be a carrier of hemophilia A.

If a person is heterozygous for a given gene, it means that the person a. is a carrier for a genetic disorder. b. is affected by the genetic disorder. c. has 2 identical alleles for the gene. d. has 2 different alleles for the gene.

d. has 2 different alleles for the gene.

The nurse is unable to flush a central venous access device and suspects occlusion. The best nursing intervention would be to a. apply warm moist compresses to the insertion site. b. try to force 10 mL of normal saline into the device. c. place the patient on the left side with the head down. d. have the patient change positions, raise arm, and cough.

d. have the patient change positions, raise arm, and cough.

The nurse tells a friend who asks him to administer his allergy shots that a. it is illegal for nurses to administer injections outside of a medical setting. b. he is qualified to do it if the friend has epinephrine in an injectable syringe provided with his extract. c. avoiding the allergens is a more effective way of controlling allergies, and allergy shots are not usually effective. d. immunotherapy should only be administered in a setting where emergency equipment and drugs are available.

d. immunotherapy should only be administered in a setting where emergency equipment and drugs are available.

Increased intraocular pressure may occur due to a. edema of the corneal stroma. b. dilation of the retinal arterioles. c. blockage of the lacrimal canals and ducts. d. increased aqueous humor production by the ciliary process.

d. increased aqueous humor production by the ciliary process.

Giving opioids to an actively dying patient who is reporting moderate to severe pain a. is ineffective. b. can lead to addiction. c. will hasten the person's death. d. is an appropriate and ethically justified nursing action.

d. is an appropriate and ethically justified nursing action.

A patient with allergic rhinitis reports severe nasal congestion; sneezing; and watery, itchy eyes and nose at various times of the year. When teaching the patient about how to control these symptoms, the nurse teaches the patient to a. avoid all intranasal sprays and oral antihistamines. b. limit the usage of nasal decongestant spray to 10 days. c. use oral decongestants at bedtime to prevent symptoms during the night. d. keep a diary of when the allergic reaction occurs and what precipitates it.

d. keep a diary of when the allergic reaction occurs and what precipitates it.

The nurse is caring for a female who had surgery 1 day ago to remove a breast mass. The patient is awaiting the pathology report. She is tearful and says that she is scared to die. The most effective nursing intervention at this point is to use this opportunity to a. review the importance of advanced directives. b. provide reassurance that everything will be fine. c. discuss healthy stress relief and coping practices. d. let her communicate about the meaning of this experience.

d. let her communicate about the meaning of this experience.

Trends in the death rates from cancer that affect health promotion programs include a. more women die of colon cancer compared to men. b. prostate cancer is the leading cause of death in men. c. breast cancer is the leading cause of cancer deaths in women. d. lung cancer is a leading cause of death among men and women.

d. lung cancer is a leading cause of death among men and women.

The patient health history and physical examination provide the nurse with information primarily to a. diagnose a medical problem. b. investigate a patient's signs and symptoms. c. classify subjective and objective patient data. d. make clinical judgments about the patient's health.

d. make clinical judgments about the patient's health.

Immediate care priorities in the first few hours for a patient with a new tracheostomy include a. encouraging early mobility. b. changing the tracheostomy ties. c. suctioning the tracheostomy hourly. d. observing for bleeding at the insertion site.

d. observing for bleeding at the insertion site.

In a type I hypersensitivity reaction the primary immunologic disorder appears to be a. binding of IgG to an antigen on a cell surface. b. deposit of antigen-antibody complexes in small vessels. c. release of cytokines used to interact with specific antigens. d. release of chemical mediators from IgE-bound mast cells and basophils.

d. release of chemical mediators from IgE-bound mast cells and basophils.

A patient has the following arterial blood gas results: pH 7.52, PaCO2 30 mm Hg, 24 mEq/L. The nurse determines that these results indicate a. metabolic acidosis. b. metabolic alkalosis. c. respiratory acidosis. d. respiratory alkalosis.

d. respiratory alkalosis.

During the postoperative care of a 76-year-old patient, the nurse monitors the patient's intake and output carefully, knowing that the patient is at risk for fluid and electrolyte imbalances primarily because a. older adults have an impaired thirst mechanism and need reminding to drink fluids. b. older adults are more likely than younger adults to lose extracellular fluid during surgeries. c. water accounts for a greater percentage of body weight in the older adult than in younger adults. d. small losses of fluid are significant because body fluids account for 45% to 50% of body weight in older adults.

d. small losses of fluid are significant because body fluids account for 45% to 50% of body weight in older adults.

An appropriate nursing intervention to help a patient with pneumonia manage thick, purulent secretions would be to a. perform postural drainage every hour. b. provide analgesics every 3 hours to promote comfort. c. administer O2 as prescribed to maintain optimal O2 levels. d. teach them how to cough effectively and expectorate secretions.

d. teach them how to cough effectively and expectorate secretions.

The most important intervention for the patient with epidemic keratoconjunctivitis is a. cleansing the affected area with baby shampoo. b. monitoring spread of infection to the opposing eye. c. regular instillation of artificial tears to the affected eye. d. teaching the patient and caregivers good hygiene techniques.

d. teaching the patient and caregivers good hygiene techniques.

While in the recovery room, a patient with a total laryngectomy is suctioned and has bloody mucus with some clots. Which nursing interventions would apply? (select all that apply) a. Notify the health care provider at once. b. Place the patient in semi-Fowler's position. c. Use a bag-valve-mask (BVM) and begin rescue breathing for the patient d. Instill 10 mL of normal saline into the tracheostomy tube to loosen secretions. e. Continue patient assessment, including O2 saturation, respiratory rate, and breath sounds.

provider at once. b. Place the patient in semi-Fowler's position. e. Continue patient assessment, including O2 saturation, respiratory rate, and breath sounds.


Kaugnay na mga set ng pag-aaral

ATOC 1060 Study Guide EXAM #1 -Giglio

View Set

Unit I-E Ethics and Values objectives

View Set

Ch. 07 Quiz: Small Business and Entrepreneurship: Economic Rocket Fuel

View Set

Complex Final Exam (Cardiac, Respiratory, Neuro, GI, Renal, Disaster)

View Set

Unit 8: Post WWII & The Cold War

View Set

Biology Section 3-2 Review: Molecules of Life

View Set